You are on page 1of 100

lOMoARcPSD|5190755

PACOP PINK Pharmacology Reviewer

Pharmacology 2 (Our Lady of Fatima University)

StuDocu is not sponsored or endorsed by any college or university


Downloaded by ;%'%SiLeNt'%; (jemribsagpatan123@gmail.com)
lOMoARcPSD|5190755

PHARMACOLOGY, TOXICOLOGY, PHARMACOKINETICS

1. Two dose-response curve response plotted on the same graph showed sigmoid curves having
the same slope and height. T curve is located to the right of the A curve. Which of the following
statements is/are correct regarding curves A an:

I. A is more effective than B


II. A is more potent than B
III. A and B have the same ceiling dose
IV. B is the graph produced when A is given with its competitive antagonist
V. B is the graph produced when A is given with its non-competitive antagonist

a. Only I and III are correct


b. Only II, III and IV are correct
c. Only II and IV are correct
d. Only III and V are correct
e. Only V is correct
2. Two dose-response curve plotted on the same graph showed sigmoid curves A and B, where A is
at the left of the graph with the height greater than B. Which of the following statements is/are
correct regarding curves A and B?

I. A and B has equal efficacy


II. A and B have equal potency
III. B is a partial agonist of A
IV. B is the graph produced when A is given with its competitive antagonist
V. B is the graph produced when A is given with its non-competitive antagonist

Downloaded by ;%'%SiLeNt'%; (jemribsagpatan123@gmail.com)


lOMoARcPSD|5190755

a. Only I and III are correct


b. Only II, III and IV are correct
c. Only II and IV are correct
d. Only III and V are correct
e. Only V is correct
3. The cumulative number of subjects who display a certain rersponse or effect to a drug is plotted
on a graph as shown below. The graph on the left represents beneficial effect while the graph on
the right represents toxic effect, what is the therapeutic index of the drug?

a.0.5 b. 1.5 c. 2.5 d.4.0


e. 5.0
4. An administration of one drug leads to a shift in the log dose-response curve of another drug to
the left a range in the maximal efficacy, this is called
a. Agonism
b. Partial agonism
c. Non-competitive antagonism
d. Competitive antagonism
e. Potentiation
5. The action of this drug is dependent on a colligative property
a. Mannitol
b. Cocaine
c. Reserpine
d. Furosemide
e. Losartan
6. Whiich of the following drug groups have mechanisms of aaction that involve binding to enzymes
as target protiens
a. Benzodiazepines
b. Barbiturates
c. Calcium channel blockers
d. Non-steroidal anti-inflammatory agents
e. Phenothiazine antipsychotics
7. Voltage-gated ion channels serve as site of actions for drugs such as
I. Diltiazem and Nifedipine
II. Lidocaine and Procaine
III. Captopril and Losartan
a. I only
b. I and II

Downloaded by ;%'%SiLeNt'%; (jemribsagpatan123@gmail.com)


lOMoARcPSD|5190755

c. III only
d. I and III
e. I, II, and III
8. Drugs with mechanism of action that involves Tubulin-binding leading to loss of function of the
structural protien include
a. Cyclosporine
b. Neostigmine
c. Colchicine
d. Glibenclamide
e. Metformin
9. What type of protien is primarily targeted by Digitalis glycosides when they exert their effect on
the heart?
a. Receptors
b. Ion channels
c. Enzymes
d. Transporters
e. Structural protiens
10. Which of the following features cahracterize nicotinic, GABA-A, and Glutamate receptors?
a. Thay control the movements of ions and out of the cell
b. Their action is mediated bysecondary messengers
c. They all have excitatory effect on the cell membrane
d. They are located on the cytoplasm
11. Which of the following drugs bind to receptors located at the cell membrane?
a. Thyroid hormone
b. Prednisone
c. Estrogen
d. Insulin
e. Vitamin D
12. Cyclic Adenosine Monophophate (cAMP) is generated as a secondary messenger by the action
of the enzyme Adenylyl Cyclase. The substrate for the synthesis of cAMP is
a. AMD
b. ADP
c. ATP
d. GTP
e. Adenosine
13. Stimulation of Beta-adrenoceptor, a G-protien-linked receptor involves a generation of a
secondary messenger known as
a. IP3
b. cGTP
c. DAG
d. cAMP
e. Calcium ions
14. What is the mechanism of action of Sildenafil, a drug popularly known as viagra?
a. Inhibits phosphodiesterase (PDE) leading to increase cAMP
b. Inhibits PDE leading to increase cGMP
c. Stimulates adenylyl cyclase leading to increase cAMP
d. Stimulates guanylyl cyclase leading to increase cGMP
e. Inhibits phosphodiesterase leading to increase cGMP

Downloaded by ;%'%SiLeNt'%; (jemribsagpatan123@gmail.com)


lOMoARcPSD|5190755

15. Drugs that activate the phospholipase C-inositol phosphate system such as alpha-adrenergic
agonists like phenylephrine primarily cause an increase in the intracellular levels of which ion?
a. Calcium
b. Magnesium
c. Sodium
d. Potassium
e. Chloride
16. Which of the following stetements characterizes competitive antagonism?
a. There is a shift of the agonist log concentration-effect curve to the right and downwards
b. There is a shift of the agonist log concentration-effect curve to the right without a change
on the slope or amplitude
c. There is an exponential relationship between dose ratio and antagonist concentration
d. There is no competition for the binding sites
e. There is a shift of the agonist log-concentration effect curve to the left without a change in
slope and amplitude
17. The minimum dose that produces the maximum achievable response is known as
a. Potency
b. ED50
c. LD50
d. Ceiling dose
e. Minimum effective dose
18. The medium dose, or the dose of the drug that produces half of the maximum achievable
response is called
a. Potency d. Ceiling dose
b. LD50 e. Binding constant
c. Therapeutic index
19. What is described by the equation given as follows:
Response=f (EN total . Xa / (Xa + Ka)
a. Relationship between occupancy of receptor and the dose of the drug
b. Relationship between occupancy of receptor and response to the drug
c. Relationship between the dose of the drug and the response to the drug
d. Relationship between the number of receptors and the dose of the drug
e. Relationship between the response to the drug and the transducer function of the tissues
20. Competitive antagonist of the receptors of Dopamine and Benzodiazepine are expected to
produced which effect?
a. Reversal of the intrinsic activity of the receptors
b. Maintain the consitutive activity of the receptors
c. Stimulate the constitutive activity of the receptors
d. Stimulate the constitutive activity of the recptorsin the absence of any ligand
e. Produce a negative efficacy
21. The PT-INR of a patient with Warfarin has been maintained at 2.5 for the last 3 months. 1 week
earlier, the patient was given Phenobarbital. What can be expected with the PT-INR of the
patient?
a. Increased PT-INR
b. Decreased PT-INR
c. Maintained PT-INR
22. Phenobarbital when given to a patient on chronic warfarin is classified as
a. Pharmacodynamic antagonist
b. Pharmacokinetic antagonist

Downloaded by ;%'%SiLeNt'%; (jemribsagpatan123@gmail.com)


lOMoARcPSD|5190755

c. Physiologic antagonist
d. Synergistic agonist
e. Additive agonist
23. Digibind which effetively terminates the action of Digitalis glycosides is classified as what type of
antagonist?
a. Pharmacodynamic antagonist
b. Pharmacokinetic antagonist
c. Physiologic antagonist
d. Chemical antagonist
e. Non-competitive antagonist
24. Omeprazole can inhibit gastric acid secretion induced by Histamine and Acetylcholine. It is thus
classified as what type of antagonist?
a. Pharmacodynamic antagonist
b. Pharmacokinetic antagonist
c. Physiologic antagonist
d. Chemivall antagonist
e. Non-competitive antagonist
25. Which of the following mechanism may explain the development of tolerance to drugs?
a. Increase in the quantity of mediators
b. Decreased metabolic degradation of the active drug
c. Decreased excretion or extrusion of drugs from cells
d. Downregulation of receptors
e. Inactivation of counterregulatory physiologic response
26. The prerequisite that drugs must be present in aqueous solution for them to be absorbed is true if
the drugs are absorbed by which mechanism?
I. Passive transport
II. Carrier-mediated transport
III. Pinocytosis
a. I only
b. I and II
c. III only
d. I and III
e. I, II, and III
27. According to the Fick’s Diffusion Equation, the rate of transport of a drug by passive diffusion is
a. Independent of the concentration gradient
b. Inversely proportional to the membrane surface area
c. Inversely proportional to the membrane thickness
d. Inversely proportional to the partition coefficient
e. Independent of the diffusion constant of drug
28. Which of the following feature common to all carrier-mediated transport processes?
a. Movement is along a concentration gradient
b. Involves non-specific binding to carriers
c. Can be characterized by allosteric inhibition
d. Displays a Michaelis-Menten Kinetics
e. Non-energy requiring
29. Which of the following substances may be absorbed significantly by pinocytosis
a. Vitamin K
b. Cyanocobalamin
c. Cardiac glycosides

Downloaded by ;%'%SiLeNt'%; (jemribsagpatan123@gmail.com)


lOMoARcPSD|5190755

d. Quarternary ammonium compounds


e. Inorganic and organic electrolytes
30. 100ml of aqueous solutionof a given drug has a concentration of 25%.50ml of Octanol was added
and after equlibration, the concentration of the drug in the aqueous layer was 5%. What is the
apparent lipid-water partition coefficient of the drug?>
a. 0.2
b. 2
c. 4
d. 6
e. 8
31. For drugs that are given constant rate, the time to reach steady rate state concentration
dependent on
a. Rate of administration
b. Biologic half-life
c. Volume distribution
d. Peak plasma concentration achieved
e. Amount of the drug in the body
32. A given drug has a biologic half life of 4 hours. How much of the drug will remain in the body 12
hours after a single dose?
a. 75%
b. 50%
c. 25%
d. 12.5%
e. 6.25%
33. In compartmental analysis of serum drug concentration versus time plots, which of the following
findings confirm a one compartment model of drug behavior?
a. An AUC above the explorated line that is less than 10% of the total AUC
b. An AUC above the explorated line that is less than 5% of the total AUC
c. Slope of the last 3 terminal points differing by more than 10% for the slope of the first 3
terminal points
d. Slope of the 3 terminal points differing by less than 20% from the slope of the 3 terminal
points
e. Cmax that is above the line explorated from the terminal points
34. What should be the rate of infusion of Theophylline if the desired plasma concentration is
10ug/mL in a 70 kg patient?Theophylline has a clearance of 0.6mL/minute per kg body weight.
a. 6ug/min
b. 4.2ug/min
c. 17ug/min
d. 420ug/min
e. 1167ug/min
35. A given drug with a half life of 8 hours is being administered at 100mg every 8 hours. What is the
amount remaining in the body after 32nd hour, just before administering the next dose?
a. 50mg
b. 75mg
c. 87.5mg
d. 93.75mg
e. 100mg

Downloaded by ;%'%SiLeNt'%; (jemribsagpatan123@gmail.com)


lOMoARcPSD|5190755

36. What is the con centration at steady state of Phenobarbital if the drug is given to a 60kg patient
as a 65mg tablet once daily with a bioavailability of 90%? The drug has volume distribution of
0.5liters/KBW and t1/2 of 100hrs
a. 0.10mg/L
b. 1.50mg/L
c. 11.70mg/L
d. 22.20mg/L
e. 48.20mg/L
37. A 47 year old female diabetic is maintained on Tolbutamide for control of her hyperglycemia. Her
fasting blood sugar is maintained at 102mg%. Three days prior to admission, she self-medicated
with Indomethacin due to a joint pain. She was admitted at the emergency room unconscious and
difficult to arouse. What laboratory finding may be consistent with the history of the patient?
a. Capillary blood glucose of 20mg/dL
b. Capillary blood glucose of 400mg/dL
c. ALT level that is 3x elevated
d. ALT level that is 6x elevated
e. Platelet level of 100,000/ul
38. A drug molecule does not possess biologic activity by itself but it converted in the body to an
active metabolite is called a/an
a. Orphan drug
b. Prodrug
c. Prototype drug
d. Parent drug
e. Metabolic drug
39. Astemizole and Terfenadine have been withdrawn from the market because of their intrinsic
cardiotoxicity. Tha risk of cardiotoxicity is increased with concominant intake of Grapefruit juiceor
Erythromycin. The explanation for this interaction is
a. Displacemnet from the protien binding of Astemizole or Terfenadine by Grapefruit juice of
Erythromycin
b. Decreased urinary excretion of Astemizole and Terfenadine
c. Inhibtion of the CYP3A4 enzyme by Grapejuice or Erythromycin
d. Increased absorption of Astemizole or Terfenadine
e. Induction of metabolism of Astemizole and terfenadineby Grapefruit juice or Erythromycin
40. Paients who are poor Debrisoquin metabolizers have decreased activity of which enzyme system
a. CYP 1A2
b. CYP 2C9
c. CYP 2D6
d. CYP 2C19
e. CYP 3A4
41. A patient has been assessed to have poor Debrisoquin metabolizing capability. Which of the
following drugs will have an increased likelihood of causing toxicity in this patient?
a. Thioridazine
b. Amiodarone
c. Acetaminophen
d. Warfarin
e. Theophylline
42. What is considered as dominant phase I metaabolic reaction?
a. Oxidation
b. Reduction

Downloaded by ;%'%SiLeNt'%; (jemribsagpatan123@gmail.com)


lOMoARcPSD|5190755

c. Hydrolysis
d. Glucuronidation
e. Acetylation
43. Chloramphenicol when given to a nenonate or premature baby causes cardiovascular collapse
manifestin as the so-called “Gray Baby Syndrome” This toxicity in such subsets of the patients is
due to what age-related biologic feature?
a. Decreased protien binding
b. Decreased urinary excretion of the metabolite
c. Deficiency of Glucuronosyl acyl transferase
d. Increased metabolism by nitro-reduction
e. Increased volume of distribution
44. Cimetidine, Ketoconazole, and Allopurinol share a common pharmacokinetic property that
predisposes patients taking other drugs concominantly with any of these agents to toxicity. What
is this property?
a. Inhibition of tubular secretion
b. Increased systemic absorption
c. Protien binding dsiplacement
d. Enzyme induction
e. Enzyme inhibition
45. Biotransformation of orally administerd drugs occuring primarily in the liver that reduces the
amount of drug finally entering the systemic circulation is often referred to as __________.
a. Enterohepatic recirculation
b. Gastric emptying
c. Elimination
d. First-pass effect
e. Enzyme induction
46. Charcoal-broiled foods, cruciferous vegetables and omeprazole induce the metabolism of drugs
like tomoxifen, acetaminophen, theophylline and warfarin. This effect is due to induction of which
CYP isoform?
a. CYP1A2
b. CYP2A6
c. CYP2C9
d. CYP2D6
e. CYP4A4
47. In a patient with hepatic encelopathy, which of the following agents may lead tosevere drug
toxicity even when given at usual doses?
a. Diazepam
b. Oral Neomycin
c. Lactulose
d. Propanolol
e. IV Gentamicin
48. Which of the following features charcterizes the autonomic nervous system
a. Single neuron transmission
b. 0
c. Effects are under volitional control
d. Conveys outpus from the CNS to the skeletal muscles
e. Terminal neurotransmitter is mainly Serotonin
49. The preganglionic fibers of the sympathetic nervous system originate from which roots?
a. Cranial nerves III, VII, IX, X

Downloaded by ;%'%SiLeNt'%; (jemribsagpatan123@gmail.com)


lOMoARcPSD|5190755

b. Sacral roots S1- S4


c. Cervical roots C1-C7
d. Thoracolumbar roots
50. What neurotransmitter is released by preganglionic fibers as a response to sympathetic
stimulation?
a. Acetylcholine
b. Norepinephrine
c. Epinephrine
d. Serotonin
e. Dopamine
51. What is the dominant ganglionic receptor found in most sympathetic fibers?
a. Alpha-1
b. Alpha-1
c. Beta-1
d. Nicotinic
e. Muscarinic
52. What is the neurotransmitter mainly released by postganglionic parasympathetic fibers?
a. Acetylcholine
b. Norepinephrine
c. Epinephrine
d. 5-Hydroxyryptamine
e. Dopamine
53. Which of the following non-adrenergic, non-cholinergic neurotransmitters is involved in the action
of the enteric nervous system?
a. ATP
b. Dopamine
c. GABA
d. Glutamate
e. Glycine
54. In synaptic neurotransmission, which of the following ions plays a significant role in
neurotransmitter release
a. Sodium
b. Potassium
c. Magnesium
d. Calcium
e. Chloride
55. The primary mechanism of neuro transmitter release is accomplished by what process?
a. Diffusion
b. Carrier-mediated
c. Active transport
d. Facilitated transport
e. Exocytosis
56. What is the amino acid precursor in the synthesis of catecholamines?
a. Phenylalanine
b. Glycine
c. Tyrosine
d. Tryptamine
e. Glutamic acid
57. In the biosynthesis of Norepinephrine, what step is considered rate-limiting?

Downloaded by ;%'%SiLeNt'%; (jemribsagpatan123@gmail.com)


lOMoARcPSD|5190755

a. Active uptake of Tyrosine


b. Conversion of tyrosine to dihydroxyphenylalanine
c. Conversion of DOPA to dopamine
d. Uptake of Dopamine in presynaptic storage vesicles
e. Conversion of Norepinephrine to Epinephrine
58. Which of the following substances exert a negative feedback control on Tyrosine Hydroxylase
which serves as the mechanis for moment-to-moment regulation of the rate catecholamine
synthesis?
a. Dopamine
b. Acetylcholine
c. DOPA
d. Epinephrine
e. Norepinephrine
59. Reserpine, an alkaloid from Rauwolfia, can lower the blood pressure and depress the CNS. What
enzyme or part of the biosynthetic pathway is inhibited by Reserpine to bring about these effects?
a. DOPA decarboxylase
b. Storage of catecholamines in the presynaptic vesicles
c. Release of Norepinephrine from presynaptic terminals
d. MAO enzymes
e. Uptake-1 mechanism
60. What is the most important mechansim of termination of effects of released Norepinephrine which
is the target of action of tricyclic antidepressants and Cocaine
a. Degradation of COMT
b. Degradation of MAO
c. Sequestration into cells
d. Reuptake into the presynaptic terminal
e. Binding to presynaptic alpha-2 receptors
61. What is the main final metabolite of Norepinephrine and Epinephrine which when abnormally
elevated is often used as a marker for the disease Pheochromocytoma?
a. Dihydroxymandelic acid
b. 3-Methoxy-4 hydromandelic acid
c. 3-Methoxy-4 hydroxyphenylglycol
d. 3,4-Dihydroxyphenylglycol
e. Normetanephrine
62. Which of the following is an expected respose to alpha-1 adrenoceptor stimulation?
a. Uterine smooth muscle relaxation
b. Gastrointestinal sphincter constriction
c. Ciliary muscle relaxation
d. Gastrointestinal wall smoothe muscle contraction
e. Platelet aggregation
63. A male patient wishes to purchase an over-the-counter tablet for colds. Which of the following
data you gathered during an initial interview of the patient should prevent you from dispensing the
medication?
I. The patient is currently taking Nifedipine GITTS
II. The patient is currently on Tranylcypromine
III. The patient is an elderly with history of urinary hesitancy, intermittency, and frequency
a. I only
b. I and II
c. III only

Downloaded by ;%'%SiLeNt'%; (jemribsagpatan123@gmail.com)


lOMoARcPSD|5190755

d. I and III
e. I, II, and III
64. Stimulation of Alpha-1 receptors leads to the formation of important secondary messengers that
promote calcium release and activation of protien kinase C. These secondary messengers
responsible for the effects of alpha-1 stimulation include which of the following?
a. Inositol triphosphate
b. Cyclic adenosine monophophate
c. Adenosine triphosphate
d. Cyclic guanosine monophosphate
e. Adenosine diphosphate
65. In which of the following clincal conditions are the agonists of the alpha-1 adrenoceptors found to
be useful?
I. Nasal congestion
II. Hypotension
III. Sympomatic bradycardia
a. I only
b. I and II
c. III only
d. I and III
e. I, II, and III
66. Which of the following effects can be attributed to alpha-2 receptor stimulation?
I. Vasodilation
II. Inhibition of catecholamine release
III. Vasoconstriction
a. I only
b. I and II
c. III only
d. I and III
e. I, II, and III
67. A given adrenergic agent induces uterine relaxation and bronchial smooth muscle relaxation.
These effects can be attributed to:
a. Alpha-1 stimulation
b. Alpha-2 stimulation
c. Beta-1 inhibition
d. Beta-1 stimulation
e. Beta-2 stimulation
68. What is the dominant adrenergic receptor in the heart?
a. Dopamine-1
b. Alpha-1
c. Alpha-2
d. Beta-1
e. Beta-2
69. Which of the following effects is/are consistent with the systemic use of the drug Clonidine?
I. Initial transient increase in blood pressure
II. “First-dose” phenomenon
III. Lowering of the blood pressure
a. I only
b. I and II
c. III only

Downloaded by ;%'%SiLeNt'%; (jemribsagpatan123@gmail.com)


lOMoARcPSD|5190755

d. I and III
e. I. II, and III
70. In terms of mechanism of action, which of the following drugs most closely resembles that of
Clonidine?
a. Phenylphrine
b. Reserpine
c. Guanethidine
d. Methyldopa
e. Amlodipine
71. Epinephrine is considered as the first line for the management of anaphylaxis and anaphylactic
shock. Which of the following explain/s the mechanism of action of Epinephrine in anaphylaxis
and anaphylactic shock?
I. Inhibition of Histamine release by beta-2 receptor activation
II. Alpha-1 activation leading to vasoconstriction
III. Beta-2 activation leading to bronchodilation
a. I only
b. I and II
c. III only
d. I and III
e. I, II, and III
72. The activity of this drug is attributed to its metabolite which is a false neurotransmitter at the
central nervous system
a. Guanethidine
b. Methyldopa
c. Propanolol
d. Labetalol
e. Phenylpropanolamine
73. Which of the following inotropic agents is most useful for paatient with acute heart failure
complicated by acute reduction in creatinine clearance?
a. Epinephrine
b. Norepinephrine
c. Dopamine
d. Dobutamine
e. Digitalis
74. In the adrenal medulla, norepinephrine is converted to epinephrine by the action of which
enzyme?
a. DOPA decarboxylase
b. Tyrosine hydroxylase
c. Cathecol-0- nethyl transferase
d. Monoamine oxidase
e. Phenylethylamine N-methyl transferase
75. As an inotropic agent, Norepinephrine (Levophed) has been shown in some small clinical trials to
be most useful in which of the following conditions?
a. Cardiogenic shock
b. Septic shock
c. Acute renal failure
d. Acute myocarditis
e. Dilated cardiomyopathy
76. Which of the following statements is/are true regarding the drug methyldopa

Downloaded by ;%'%SiLeNt'%; (jemribsagpatan123@gmail.com)


lOMoARcPSD|5190755

I. Is current clinical use is in the management of hypertension in pregnancy


II. It is associated with warm antibody type of immunohemolytic anemia
III. Toxic doses are associated with hepatotoxicity
a. I only
b. I andII
c. III only
d. I and III
e. I, II, and III
77. What are known clinical uses beta-2 agonists?
I. In the management of premature labor
II. As part of the interventions employed for hyperkalemia
III. Management of bronchial asthma
a. I only
b. I andII
c. III only
d. I and III
e. I, II, and III
78. Which of the following agents is classified as a selective direct-acting beta-2 agonist?
a. Propanolol
b. Mathoxamine
c. Isoxsuphrine
d. Prazosin
e. Ephedrine
79. During the conduct of ACLS, what is the recommended dosing ofor Epinephrine in adults?
a. 1mg every 3-5 minutes
b. 3mg every 3-5 minutes
c. Escalating dose, staring at 1mg then increasing by 1mg every 3-5 minutes
d. 10ml of a 1:1000 solution every 3-5 minutes
80. Which of the following is an accepted indication for Epinephrine?
a. Bronchodilator fo intermittent bronchial asthma
b. Local vasoconstrictor
c. Anti-arrythmic for supraventricular tachycardia
d. Tocolytic agent for premature labor
e. Inotropic for cardiogenic shock
81. A dominant alpha-1 agonist effect is expected when Epinephrine is given as an IV infusion at
what doses?
a. <10ng/kg/min
b. 10-50ng/kg/min
c. 10-80ng/kg/min
d. At least 120ng/kg/min
82. What is the clinical use of the Dopamine-1 agonist Fenoldopan?
a. Chronic treatment of Congestive Heart Failure
b. Acute treatment of Pheochromocytoma
c. Acute treatment of Hypertension
d. Chronis treatment of Supraventricular Tachycardia
e. Acute treatment of Congestive Heart failure
83. Which of the following adrenergic agents is/are most useful for hypodynamic septic shock with
hypotension
I. Dopamine

Downloaded by ;%'%SiLeNt'%; (jemribsagpatan123@gmail.com)


lOMoARcPSD|5190755

II. Dobutamine
III. Norepinephrine
a. I only
b. I andII
c. III only
d. I and III
e. I, II, and III
84. At what dose should Dopamine be given as IV infusion to produce a predominantly dopaminergic
effect?
a. 0.5 ug/kg/min
b. 5-10ug/kg/min
c. 10-20ug/kg/min
d. 20-50ug/kg/min
e. >50ug/kg/min
85. Guanfacine and Guanabenz are primarily agonists at what receeptors?
a. Muscarinic-1
b. Nicotinic-N
c. Presynaptic alpha-2
d. Postsynaptic alpha-2
e. Alpha-1
86. Which of the following is/are correct clinical indication for the drug Terbutaline
I. Subcutaneously given as management of recalcitrant acute exacerbations of bronchial
asthma
II. To increase heart rate in symptomatic bradycardia
III. Adjunct treatment of malignant hypertension
a. I only
b. I andII
c. III only
d. I and III
e. I, II, and III
87. Which of the following drugs primarily stimulates beta-1 receptors?
a. Dobutammine
b. Ritodrine
c. Yohimbine
d. Phenoxybenzamine
e. Oxymetazoline
88. A new drug is currently being developed whose mechanism of action is stimulation of the beta-3
receptors. In which condition will this drug have most likely usefulness?
a. Obesity
b. Bronchial asthma
c. Hypertension
d. Shock
e. Acute renal failure
89. A patient complains of muscle weakness after several doses of Salbutamol nebulization. This
condition can be attributed to _______________?
a. Hypokalemia induced by Salbutamol therapy
b. Hypocalcemia induced by Salbutamol therapy
c. Metabolic alkalosis due to bronchial asthma
d. Decreased firing of neuromuscular junction fibers as a direct effect of Salbutamol

Downloaded by ;%'%SiLeNt'%; (jemribsagpatan123@gmail.com)


lOMoARcPSD|5190755

e. Respiratory alkalosis due to bronchial asthma


90. Which of the following beta-blockers also has peripheral vasodilating effect similar to Prasozin?
a. Propanolol
b. Betaxolol
c. Nadolol
d. Bisoprolol
e. labetalol
91. Which of the following beta-blockers is considered cardio selective?
a. Labetalol
b. propanolol
c. pindolol
d. Esmolol
e. Timolol
92. Rebound tachycardia and hypertension are expected complications of abrupt withdrawal of beta-
blocker therapy. Which of the followingagents has the least likelihood of causing rebound effects
when withdrawn?
a. Labetalol
b. Propanolol
c. Pindolol
d. Metropolol
e. Timolol
93. Which of the following beta-blockers has the shortest duration of action (half-life = 10 minutes)
making is useful for administration as continuous IV infusion with rapid achievemnet of a steady
state concentration
a. Metoprolol
b. Esmolol
c. Atenolol
d. Propanolol
e. Pindolol
94. Which of the following beta-blockers has/have been proven in clinical trials to be useful in the
management os stable congestive heart failure
I. Carvedilol
II. Bisoprolol
III. Metoprolol
a. I only
b. I andII
c. III only
d. I and III
e. I, II, and III
95. In which of the following conditions are beta-blockers found to be useful?
I. Hypothyroidism
II. Raynaud’s Phenomenon
III. Initial therapy in Pheochromocytoma
a. I only
b. I andII
c. III only
d. I and III
e. I, II, and III

Downloaded by ;%'%SiLeNt'%; (jemribsagpatan123@gmail.com)


lOMoARcPSD|5190755

96. Among diabetic patients using insulin or oral hypoglycemic agents (OHAs) why are beta-blockers
are used with special caution if not totally avoided?
a. Beta-blockers can induce hyperglycemia
b. Beta-blockers can mask hypoglemic symptoms
c. Beta-blockers can attenuate effect of insulin and OHAs
d. Beta-blockers can induce insulin or OHA failure
e. Beta-blockers can increase the risk of renal failure
97. In which subset/s of patients are beta-blockers used with caution if not avoided?
I. Patients with bronchospastic disease like COPD and bronchial asthma
II. Patients with peripheral vascular disease especially vasospastic type
III. Patients with recent myocardial infarction
a. I only
b. I andII
c. III only
d. I and III
e. I, II, and III
98. A 44-year old patient is started a week ago on the following medications: Metoprolol 100mg BIB,
Diltiazem 30mb tablet BID and Indapamide 1.5 mg OD for poorly controlled hypertension and
chronic stable angina pectoris. She is now admitted at the ICU for dizziness and two episodes of
syncope. Her heart rate was 45 beats/min. What course of action would you suggest?
a. Continue her medication and suggest for a 12-lead ECG
b. Stop Metoprolol and request for a 12-lead ECG
c. Stop Diltiazem and request for a 12-lead ECG
d. Stop indapamide and Diltiazem and request for a 12-lead ECG as well as serum
potassium measurement
e. Stop Metoprolol and Diltiazem and request for a 12-lead ECG
99. Methylphenidate and Dextroamphetamine are centrally acting sympathomimetics which have
found use in which of the following conditions?
a. Manic-depression
b. Alcohol withdrawal
c. Attention deficit hyperactivity disorder
d. Generalized anxiety disorder
e. Hypotensive epidoses
100. Which of the following effects is are consistent with direct stimulationof the muscarinic
receptors?
I. Peripheral vasodilation
II. Relaxation of the ciliary muscles
III. Negative dromotropic effect
a. a. I only
b. I andII
c. III only
d. I and III
e. I, II, and III
101. The effect of muscarinic receptor activation in tissues like the exocrine glands and the
extravascular smooth muscles have been linked with the generation of which second messenger
molecule?
a. IP3
b. cAMP
c. cGMP

Downloaded by ;%'%SiLeNt'%; (jemribsagpatan123@gmail.com)


lOMoARcPSD|5190755

d. ATP
e. Adenosine
102. What is thw physiologic event that follows stimulation of the NM type cholinergic
receptors?
a. Opening of inward chloride channels
b. Opening of outward potassium channels
c. Opening of inward sodium channels
d. Opening of the outward calcium channels
e. Opening of the outwad sodium channels
103. Bethanecol is clinically used for which of the following conditions?
a. Urinary retention in neurogenic bladder
b. Intestinal obstruction
c. Chronic glaucoma
d. Supraventicular tachycardia
e. Congestive heart failure
104. What is/are the clinical use/s of Edorphonium (Tensilon)?
I. Reverse the effect of neuromuscular blockers after surgery
II. Treatment of overdose of atropine
III. Management of organophosphate poisoning
a. I only
b. I andII
c. III only
d. I and III
e. I, II, and III
105. Ambenonium, neostigmine, and pyridostigmine are most commonly used for which
condition?
a. Atropine overdose
b. Myasthenia gravis
c. Bronchial asthma
d. COPD
e. ADHD
106. Tacrine, Donepezil, Rivastigmine and Galantamine are indicated for which of the
following condition?
a. Supraventricular tachycardia
b. Curare-induced muscle paralysis
c. Accomodative esotropia
d. Alzheimer’s disease
e. Glaucoma
107. Which of the following agents maybe used for the management of glaucoma?
I. Piocarpine-Physostigmine combination therapy
II. Ecothiophate
III. Demecarium
a. I only
b. I andII
c. III only
d. I and III
e. I, II, and III
108. Which of the following findings maybe consistent with an overdose of or accidental
exposure to an organophosphate

Downloaded by ;%'%SiLeNt'%; (jemribsagpatan123@gmail.com)


lOMoARcPSD|5190755

a. Constipation
b. Mydriasis
c. Peeripheral vasodilation
d. Emesis
e. Decreased sweating
109. Which of the following effects can be attributed to atropine?
I. Ciliary muscle contraction
II. Contraction of the papillary constrictor muscles
III. Blockade of vagal slowing of the heart
a. I only
b. I andII
c. III only
d. I and III
e. I, II, and III
110. In comparison to other aantimuscarinic agents, Scopolamineis expected to exert
significant CNS effects due to:
a. Its being a tertiary amine
b. Its being a quarternary amine
c. The presence of a benzene ring structure
d. The tropic acid structureits isomeric structure
111. Ipratropium, Oxytropium, and Tiotropium and antimuscarinic agents that are most useful
as
a. Mydriatics
b. Cycloplegic agents
c. Relaxants of bronchial smooth muscles
d. Inhibitors of involuntary muscle contractions
e. Inhibitors of bronchial gland secretions
112. Oxynutinin, Dicyclomine, and Glycopyrrolate are clinically useful for which of the following
conditions?
a. Cholinomimetic agent overdose
b. Gastrointestinal hypermotility and urinary incontinence
c. Angina due to coronary artery vasospasm
d. Motion sickness and Parkinsonism
e. COPD and bronchial asthma
113. In most patients admitted to atropine overdose, most poison control experts recommend
which of the following measures as primary management?
I. Temperature control with cooling blankets
II. Physostigmine as initial therapy
III. Diazepam for seizure control
a. I only
b. I andII
c. III only
d. I and III
e. I, II, and III
114. Poisoning resulting from ingestion of high dose quarternary antimuscarinic agents can
be best managed with which of th following agents?
a. Physostigmine
b. Neostigmine
c. Policarpine

Downloaded by ;%'%SiLeNt'%; (jemribsagpatan123@gmail.com)


lOMoARcPSD|5190755

d. Atropine
e. Scopolamine
115. Which of the following is/are important contraindication to the use of antimuscarinis
agents or drugs with atropine-like properties?
I. History of prostatic hyperplasia
II. Glaucoma
III. Hypertension
a. I only
b. I andII
c. III only
d. I and III
e. I, II, and III
116. Trimethaphan is expected to produce which of the following effects?
a. Excessive sweating
b. Pupillary constriction
c. Diarrhea
d. Urinary incontinence
e. Hypotension
117. What is the mechanism of skeletal muscle paralysis produces by Succinylcholine?
a. Competitively inhibits binding of acetylcholine to the NM receptors
b. Increased inward chloride conductance leading to hyperpolarization of the neuromuscular
endplate membrane
c. Sustained depolarization of the motor end plate making it unesponsive to other impulses
upon repolarization cannot easily depolarized
d. Repeated short depolarizations of the motor end plate leading to prolonged state of
sodium channel inactivation
e. Irreversible and non-competitive inhibition of the NM receptors in both the ganglia and
motor end plate with attenuation of depolarizing impulses
118. Which of the following findings is consistent with the skeletal muscle relaxant effect of
Pancuronium and Rocuronium?
a. Effect is enhanced with administration of succinylcholine
b. Effect is reduced with co-administration of tubocurarine
c. Initially cause depolarization of the motor end plate leading to fassiculations
d. Effect is antagonized by Neostigmine
e. Sustained response to tetanic stimulation
119. Which of the following inhilational anesthetics produces greatest augmentation of the
effect of neuromuscular blockers?
a. Isoflurane
b. Sevoflurane
c. Halothane
d. Desflurane
e. Nitrous oxide
120. Anaphylactoid reaction may be seen as a side effect with which of the following
neuromuscular blockers?
a. Pancuronium
b. Vecuronium
c. Gallamine
d. Tubocurarine
e. Rocuronium

Downloaded by ;%'%SiLeNt'%; (jemribsagpatan123@gmail.com)


lOMoARcPSD|5190755

121. Which of the following statements correctly describes the side effects of succinylcholine?
I. Negative chonotropic and inotropic effects which can be attenuated by atropine
II. Increase risk of emesis among diabetics and obese patients
III. Hyperkalemia when given to burn or trauma patients or patients with close head injury
a. I only
b. I andII
c. III only
d. I and III
e. I, II, and III
122. Which of the following mechanism may explain the ability of beta-blockers to reduce
blood pressure in hypertension?
a. Increased compliance of the capacitance vessels
b. Relaxation of the primary resistance vessels
c. Inhibition of renin release
d. Direct inhibition of Angiotensin II receptors
e. Decrease in total body stores of sodium
123. Which of the following equations describe the factors that contribute to blood pressure?
a. BP=SV/SVR
b. BP=COxSVR
c. BP=SVxHR
d. BP=CO/SVR
e. BP=SVxSVR
124. Which of the following mechanism explains the blood pressure-lowering effect of thiazide
diuretics?
I. Reduction in blood volume and cardiac output
II. Increased vascular compliance
III. Decreased proximal tubular reabsorption of water and sodium
a. I only
b. I andII
c. III only
d. I and III
e. I, II, and III
125. Which of the following drugs inhibit enzyme carbonic anhydrase?
a. Dichlorphenamide
b. Tolazamide
c. Astemizole
d. Selegiline
e. Donepezil
126. What is the primary mechanism of action of the drug Indapamide?
a. Inhinition of the 2Cl-Na-K cotransporters in the convoluted tubule
b. Inhibition of the Na-Cl contransporters at the convoluted tubule
c. Inhibition of the Na-Cl cotransporters at the collecting duct
d. Competitive inhibition of aldosterone binding to its receptors
e. Competitive inhibition os vasopressin binding to V2 receptors
127. Which side effect of chlorthalidone is prominent especially among the elderly when
Chlorthalidone is given at a dose of at leats 50mg/day and significantly reduced when doses are
maintained below 50mg/day?
a. Hyperuricemia
b. Hyperglycemia

Downloaded by ;%'%SiLeNt'%; (jemribsagpatan123@gmail.com)


lOMoARcPSD|5190755

c. Increased in serum LDL


d. Allergic reactions
e. Hyponatremia
128. What is the rationale behind the combination of amiloride and Hydrochlorothiazide
(HCTZ)
a. Minimize hyperglycemia due to HCTZ
b. Reduce risk of renal stone formation with HCTZ
c. Reduce hyperkalemia with amiloride
d. Minimize bicarbonaturia with HCTZ
e. Minimize hypokalemia due to HCTZ
129. The action of the drug Eplerenone most closely resembles which of the following agents?
a. Spironolactone
b. Amiloride
c. Triamterence
d. Metolazone
e. Quinethazone
130. Which of the following agents exhibits pharmacodynamic interaction with the thiazide
diuretics leading to a reduced diuretic effect?
a. Furosemide
b. Amiloride
c. Enalapril
d. Hydralzine
e. Digoxin
131. Which of the following mechanism explain the effects of the drug Torsemide and
Bumetanide?
a. Inhibition of carbonic anhydrase at the proximal convoluted tubule
b. Inhibition of the Na-K-2Cl co-transporter at the thick ascending limb of the Loop of Henle
c. Inhibition of the Na-Cl co-transporter at the thin ascending limb of the Loop of Henle
d. Inhibition of the Na-Cl co-transporter at the distal convoluted tubule
e. Inhibition of the Na-Cl co-transporter at the distal tubule and collecting duct
132. Which of the following is/are accepted indication/s for the drug Spironolactone?
I. Adjunct in the management of CHF
II. Conn’s Syndrome
III. Nephrogenic Diabetis Insidipus
a. I only
b. III only
c. I and II
d. II and III
e. I, II, and III
133. Which of the following is/are accepted indication/s for the drug Furosemide?
I. Infusion-diuresis in hypercalcemia
II. Oliguric acute renal failure
III. Acute pulmonary edema
a. I only
b. III only
c. I and II
d. II and III
e. I, II, and III

Downloaded by ;%'%SiLeNt'%; (jemribsagpatan123@gmail.com)


lOMoARcPSD|5190755

134. Which of the folling agents is primarily indicated for the management of increased
intracranial pressure
a. Mannitol
b. Acetazolamide
c. Furosemide
d. Chlorthalidone
e. Bumetanide
135. Which of the following agents is most useful for the management of nephrolithiasisndue
to idiopathic hypercalciuria?
a. Ethacrynic acid
b. HCTZ
c. Dorzolamide
d. Vasopressin
e. Triamterene
136. Which of the following statements correctly describes the characteristics of Indapamide
as a drug for hypertension?
I. Its diuretic effect is self-limiting, and may not account for its hypertensive effect after
about 2-4 weeks of therapy
II. Long-term use leads to improved blood vessel compliance
III. Has modest vasodilating property
a. I only
b. II only
c. I and III
d. II and III
e. I, II, and III
137. Which of the following agents maybe associated with ototoxicity which may have
increased risk of occuring when patient also recieve aminoglycoside antibiotic?
a. Furosemide
b. Chlorthalidone
c. Dorzolamide
d. Amiloride
e. Spironolactone
138. Which of the following mechanisms explains best the action of drug methyldopa?
a. The molecule releases norepinephrine in the presynaptic vesicles and is released by
nerve stimulation to interact with postsynaptic adrenergic receptors
b. The molecule is converted first to alpha-methyldopamine and alpha-
methylnorepinephrine which replace norepinephrine in the presynaptic vesicles and are
released by nerve stimulation to interact with postsynaptic adrenergic receptors
c. The molecule is converted first to alpha-methyldopamine and alpha-norepinephrine which
stimulate central alpha receptors
d. The molecule is converted first to alpha-methyldopamine and alpha-norepinephrine which
inhibit central alpha receptors
e. The molecule directly inhibits peripheral alpha and beta receptors
139. What is the most frequent side effect of methyldopa which can be seen particularly at the
onset of treatment?
a. Sedation
b. Depression
c. Extrapyramidal signs
d. (+) coomb’s test

Downloaded by ;%'%SiLeNt'%; (jemribsagpatan123@gmail.com)


lOMoARcPSD|5190755

e. Galactorrhea
140. A 45 years old female in clonidine 1.0mg/day for thee last ffour weeks was brought to the
hospital for 1 day history of nervousness, tachycardia and abrupt increase in her blood pressure
to 180/130mmHg. Her last intake of clonidine was last 2 days prior to admission. Which of the
following is/are appropriate interventions that you can suggest for the patient?
I. Start patient on Propanolol 50mg BID
II. Re-institute Clonidine 1.0mg/day
III. Start Labetalol 200mg/day
a. I only
b. III only
c. I and II
d. II and III
141. Which of the following side effects is/ are common to the three drugs: Hydralazine,
Diazoxide, and Minoxidil?
a. Hypertrichosis
b. Glucose intolerance
c. Reflex tachycardia
d. Lupus-like side effect
e. Dyslipidemia
142. In which subset of patients should Hydralazine be avoided or used with caution?
a. Patients with arrythmias
b. Patient with hypertension
c. Patients with DM
d. Patients with Ischemic Heart disease
e. Patients with CHF
143. Which of the following drugs if combined with Minoxidil may minimize latter drug’s side
effect?
a. Hydralazine+HCTZ
b. Guanethidine+Reserpine
c. Metoprolol+Chlorthalidone
d. Amlodipine
e. Amlodipine+Prazosin
144. In the management of Pparoxysmal hypertensionin pheochromocytoma, which of the
following drugs is the most appropriate initial therapy?
a. Phentolamine
b. Metoprolol
c. Guanfacine
d. Reserpine
e. Clonidine
145. Orthostatic hypotension and syncope are manifestations of a phenomenon associated
with the first dose of which agent?
a. Labetalol
b. Methyldopa
c. Fenoldopam
d. Irbesartan
e. Doxazosin
146. The mechanism of action of the drug clonidine most closely resembles that of
______________?
a. Methyldopa

Downloaded by ;%'%SiLeNt'%; (jemribsagpatan123@gmail.com)


lOMoARcPSD|5190755

b. Propanolol
c. Carvedilol
d. Guanethidine
e. Reserpine
147. Constipation, Urinary retention, tachycardia, mydriasis and anhidrosis are expected
effects based on the mechanism of action of which of the following drugs?
a. Clonidine
b. Trimethaphan
c. Reserpine
d. Sodium nitropusside
e. Nicardipine
148. Which of the following drugs may be used safely for the control of hypertension in
pregnant patients with pre-eclampsia?
I. Magnesium Sulfate
II. Methyldopa
III. Hydralazine
a. I only
b. II only
c. I and II
d. II and III
e. I, II, aand III
149. Which of the following mechanisms explains the action of the drug Sodium Nitropusside?
a. Activation of guanylyl cyclase with increase of cGMP
b. Release of N20 with subsequent increase in cAMP
c. Inhibition of the secretion of renin
d. Stimulation of A1 receptors
e. Increase conductance of outward potassium channels
150. Which of the following agents is pure anteriolar vasodilaator?
a. Prazosin
b. Sodium Nitropusside
c. Diazoxide
d. Phentolamine
e. Pindolol
151. Which of the following agents can be used for hypertensive emergencies
I. Enalaprilat
II. Diazoxide
III. Sodium nitropusside
a. I only
b. II only
c. I and II
d. II and III
e. I, II, and III
152. The decrease in arterial tone due to vasorelaxant action of sodium nitropusside is
associated with
a. Increase in smooth muscle cGMP levels
b. Increase in smooth muscle cAMP levels
c. A decrease in calcium entry through L-type channels
d. A local anethetic effect on smooth muscle cells
e. Stimulation of inward K-channel

Downloaded by ;%'%SiLeNt'%; (jemribsagpatan123@gmail.com)


lOMoARcPSD|5190755

153. Whgich of the following effects can be attributed to Angiotensin II?


I. Stimulation of release of nitric oxide
II. Stimulation of synthesis and release of Aldosterone
III. Stimulation of release of Norepinephrine
a. I only
b. II only
c. I and II
d. II and III
e. I, II, and III
154. What is the primary advantage of Angiotensin II receptor blockers over the ACE-
Inhibitors?
a. Lesser incidence of coughs
b. Absence of risk interstitial nephritis
c. Safe for pregnant patients with hypertension
d. Faster onset of action
e. Lesser incidence of hyperkalemia
155. Which of the following ACE-Inhibitors require hydrolysis to the active metabolite/s in the
body to produce effects?
I. Enalapril
II. Ramipril
III. Perindopril
a. I only
b. II only
c. I and II
d. II and III
e. I, II and III
156. In which of the following conditions are ACE-inhibitors found to be clinically useful as
first-line treatment?
I. Angina pectoris
II. CHF
III. Diabetic Nephropathy
a. I only
b. II only
c. I and II
d. II and III
e. I, II, and III
157. The efficacy of ACE-inhibitors in the management of hypertension may be reduced in the
presence of certain drugs such as ___________?
a. Ibuprofen
b. Felodipine
c. Indapamide
d. Eprenolone
e. Hydralazine
158. In which subset of patients which are ACE-inhibitors avoided as treatment option?
I. Patients with bilateral renal artery stenosis
II. Patients with hyperkalemia
III. Pregnant patients in their 2nd and 3rd trimesters of pregnancy
a. I only
b. II only

Downloaded by ;%'%SiLeNt'%; (jemribsagpatan123@gmail.com)


lOMoARcPSD|5190755

c. I and II
d. II and III
e. I, II, and III
159. Which of the following calcium channel blockers has greater activity on calcium channels
in myocardial tissues than calcium channels in the vascular smooth muscles?
a. Diltiazem
b. Verapamil
c. Nifedipine
d. Amlodipine
e. Felodipine
160. What is the mechanism of action of the diydropyridine calcium channel blockers?
a. Inhibit the L-type calcium channels in the arteriolar smooth muscles
b. Inhibit the L-type calcium channels in the anteriolar and venous smooth muscles
c. Equally inhibit the L-type calcium channels in the vascular and cardiac tissues
d. Inhibit the S-type calcium channels in the anteriolar smooth muscles
e. Inhibit the S-type calcium channels in the anteriolar and venous smooth muscles
161. In the long-term management of hypertensiion, which of the following agents is/are
considered as appropriate maintenance therapy?
I. Regular Nifedipine capsules
II. Enalapril tablets
III. Indapamide tablets
a. I only
b. II only
c. I and II
d. II and III
e. I, II, and III
162. What is the most common side effect of Enalapril?
a. Angioedema
b. Interstitial nephritis
c. Coughs
d. Hypotension
e. Drug fever
163. Which of the following anti-hypertensive agents is the most appropriate for elderly male
patients with benign prostatic hyperplasia?
a. Prazosin
b. Amlodipine
c. Metoprolol
d. HCTZ
e. Enalapril
164. Which of the following anti-hypertensive agents may be more appropriate to use for
diabetic patients with evidence of renal disease?
a. Prazosin
b. Amlodipine
c. Metoprolol
d. HCTZ
e. Enalapril
165. Chinese patients are most likely to be more sensitiveto effects of which of the following
drugs for hypertension that lower doses may have to be used?
a. Prazosin

Downloaded by ;%'%SiLeNt'%; (jemribsagpatan123@gmail.com)


lOMoARcPSD|5190755

b. Amlodipine
c. Metoprolol
d. HCTZ
e. Enalapril
166. In the management of hypertensive emergencies, which of the following goals or
interventions is/are correct?
I. Rapid normalization of the blood pressure within 24 hours
II. Lowering of the mean arterial pressure by about 25%
III. Initiation of oral hypertensives early as soon adequate blood pressure lowering has been
achieved
a. I only
b. II only
c. I and II
d. II and III
e. I, II, and III
167. Which of the following are important considerations regarding the proper handling of
Nitroglycerin tablets
I. Formulations used in the medicine are explosive
II. Sublingual tablets lose their potency when stored in plastic containers
III. These are sensitive to light
a. I only
b. II only
c. I and II
d. II and III
e. I, II, and III
168. Which of the availabe nitrovasodilators has a bioavailabilty of about 100%?
a. Isosorbide mononitrate
b. Isosorbide dinitrate
c. Amyl nitrite
d. Glyceryl trinitrate
e. None aqs all undergo significant first pass effect when given orally
169. Which of the following statements most closelydescribes the primary mechanism of
action of the nitrovasodilator?
a. Denitrification by glutathione S-transferase leads to release of nitrite ion which is then
converted to nitric oxide, a molecule that activates adenylyl cyclase leading to increase
cAMP
b. Denitrification by glutathione S-transferase leads to release of nitrite ion which is then
converted to nitric oxide, a molecule that activates guanylyl cyclase leading to increase
cGMP
c. Nitrate ion released upon denitrification directly stimulas outward K-channels leading to
hyperpolarization of the vascular smooth muscles
d. Nitrate ion released upon denitrification directly stimulas outward K-channels leading to
hyperpolarization of the vascular smooth muscles
e. Nitrate ion released upon denitrification stimulates the production of prostacylin leading to
vasodilation
170. Which of the following tissues exhibit the greatest sensitivity to nitrovasodilators at the
lowest effective doses?
a. Arteries
b. Arterioles

Downloaded by ;%'%SiLeNt'%; (jemribsagpatan123@gmail.com)


lOMoARcPSD|5190755

c. Capillaries
d. Veind
e. Myocardial tissues
171. Which of the following statements correctly explains the primary mechanism of relief of
angina pectoris with the use of nitrovasodilators when given at usual doses?
a. Peripheral dilation of the veins leading to reduction in cardiac preload and myocardial
oxygen demand
b. Vasodilation of epicardial coronary arteries leading to improved oxygen delivery to
myocardial tissues
c. Increase in diastolic perfusion time leading to improved perfusion to ischemic
myocardium
d. Decrease total peripheral resistance with arteriolar vasodilation
e. Decrease in myocardial contractility leading to reduction in myocardial oxygen demand
172. Which of the following agents classified as nitrovasodilators is useful in the management
of cyanide poisoning?
a. Sodium nitrite
b. Amyl nitrite
c. Sodium thiosulfate
d. Nitroglycerin
e. Isosorbide mononitrate
173. Which of the following nitrovasodilators may induce conversion of hemoglobin to
methemoglobin (methemoglobinemia) which can lead to cyanosis?
a. Nitroglycerin
b. Isosorbide mononitrate
c. Isosorbide dinitrate
d. Tetrathyl trinitrate
e. Amyl nitrite
174. This side effect of nitrovasodilators is explained in part by the depletion of sulfhydryl
moieties in vascular smooth muscles with continuous use of the drugs, and may be partly
reversed bysulfhydryl-regenerating compunds.
a. Tolerance
b. Increased intracranial pressure
c. Reflex tachycardia
d. Methemoglobenia
e. carcinogenicity
175. Which of the following agents is/are considered useful as single agent in the acute
treatment of vasospastic angina pectoris?
I. Nitrovasodilators
II. Beta-blockers
III. Calcium channel blockers
a. I only
b. II only
c. I and II
d. II and III
e. I, II, and III
176. What is the primary limitation with the use of Dihydropyridine Calcium Channel Blockers
in the management of angina pectoris?
a. Tendency to reduce cardiac preload
b. Tendency to cause reflex tachycardia

Downloaded by ;%'%SiLeNt'%; (jemribsagpatan123@gmail.com)


lOMoARcPSD|5190755

c. Tendency to induce reflex vasospam when stopped


d. Tendency to cause peripheral edema
e. Tendency to reduce cardiac end-diastolic pressure
177. What is the most important role of beta-blockers in the management of angina pectoris
a. Causes coronary artery vasodilatation thus useful in the management of acute anginal
attacks
b. Dilates the epicardial coronary vessels thus useful in patients with ongoing prinzmetal
angina
c. Reduces myocardial oxygen demand by reducing the total peripheral resistance making it
useful for chronic stable angina pectoris
d. Reduces myocardial oxygen demand by reducing myocardial perfusion time making it
useful for chronic stable angina pectoris
e. Reduces myocardial oxygen demand by decreasing myocardial perfusion time making it
useful for all types of angina pectoris
178. Which of the following drugs or drug combination is/are useful in the management of
chronic stable angina pectoris?
I. Regular release Nifedipine capsule
II. Metoprolol tablet
III. Metoprolol tablet + Diltiazem tablet
a. I only
b. II only
c. I and III
d. II and III
e. I, II, and III
179. What is the principal mechanism of action of cardiac Glycosides?
a. Stimulate the release of calcium for the sarcoplasmic reticulum leading to release in an
increase intracellular calcium
b. Inhibition of the cell membrane outward calcium transporter preventing calcium extrusion
c. Stimulation of inward calcium transporter leading to an increase in intacellular calcium
d. Inhibition of the Na-K-ATPase leading to increase in intracellular sodium and preventing
calcium extrusion
e. Inhibition oh the Na-Ca exchanger in the sarcoplasmic reticulum leading to increase in
intracellular calcium
180. Which of the following statements on pharmacokinetic parameters must be considered
for patients taking Digoxin?
a. In about 10% of patients, use of antibiotics can lead to increased oral bioavailability
b. Renal clearance of the drug is inversely proprtional to the creatinine clearance
c. Half-life is long, at about 168 hours due in part to extensive enterohepatic recirculation
d. Hepatic metabolism accounts for more than 80% of the elimination of the drug
e. It is primarily distributed within the intravascular space due to its high protein binding
181. Which of the following is/are mechanical effects of digoxin on myocardial tissues
I. Increase in contractility of myocardial tissues
II. Increase in conduction velocity in the AV node
III. A positive inotropic effect
a. I only
b. II only
c. I and III
d. II and III
e. I, II, and III

Downloaded by ;%'%SiLeNt'%; (jemribsagpatan123@gmail.com)


lOMoARcPSD|5190755

182. Which of the following is/are expected electrical effect/s of digoxin on myocardial
tissues?
I. Decrease in the refractory period of atrial muscles
II. Increase in the refractory period of the AV node
III. A positive inotropic effect
a. I only
b. II only
c. I and III
d. II and III
e. I, II, and III
183. Which of the following condition may augment the effect of digitalis glycosides in
myoccardial tissues?
I. Concurrent use of captopril
II. Reduced oxygenation of myocardial tissues
III. Concurrent administration of calcium gluconate
a. I only
b. II only
c. I and III
d. II and III
e. I, II, and III
184. Which of the following is/are the primary indication/s of Digitalis Glycosides?
I. First line drug for the initial management of CHF
II. Useful in controlling the rate of ventricular response in atrial fibrillation
III. Useful in pregnant patients with CHF
a. I only
b. II only
c. I and III
d. II and III
e. I, II, and III
185. What is the major mechanism of interaction between Digoxin and Quinidine?
a. Displacement of digoxin from tissue binding site
b. Decreased hepatic metabolism of digoxin
c. Decresed renal clearance of digoxin
d. Increase binding of digoxin with ion transporters in the myocardium
e. Increase intestinal absorption of digoxin
186. A 50 years old male patient with CHF was started on the following medications: digoxin
0.25 mg OD, Hydralazine 10mg BID, furosemide 20mg OD, and ISMN 60mg OD. 1 week after
initiating therapy, the patient was admitted for blurring of vision and palpitation. He feels
nauseated and has problems discriminating colors. Which of the following sourses of action is
appropriate?
a. Suggest discontinuing Hydralazine and starting patient on amlodipine
b. Suggest giving of metoclopramide for the nausea and examining the probable eye
problem
c. Suggest starting patient on beta-blocker therapy
d. Suggest discontinuing digoxin and furosemide and checking for serum K levels for
necessary correction
e. Suggest discontinuing all medications and starting patient instead on an ACE inhibitor,
vasodilator and a thiazide

Downloaded by ;%'%SiLeNt'%; (jemribsagpatan123@gmail.com)


lOMoARcPSD|5190755

187. Which of the following intervention may be appropriate in the management of severe
cases of digitalis intoxicationsuch as in children and in suicidal overdose?
I. Administration of digoxin immune fab
II. Aadministration of KCl
III. Administration of MgSO4
a. I only
b. II only
c. I and III
d. II and III
e. I, II, and III
188. What is considdered as the toxic plasma concentration of digoxin?
a. >0.5mg/mL
b. >150mgng/mL
c. >10ng/mL
d. >5ng/mL
e. >2ng/mL
189. What is the primary use of drugs like Inamirone and Milrinone?
I. Alternative to digoxin in patients with chronic CHF
II. Management for acute heart failure
III. Management of exacerbation of chronic CHF
a. I only
b. II only
c. I and III
d. II and III
e. I, II, and III
190. What is the mechanism of action of Inamrinone and Milrinone in the management of
heart failure?
a. Increase the levels of cAMP by activating the enzyme adenylyl cyclase
b. Increase the levels of cAMP by inhibiting the enzyme phosphodiesterase III
c. Increase the levels of cGMP by inhibiting the enzyme guanylyl cyclase
d. Inhibit the outward calcium flux leading to increased intracellular calcium
e. Primarily cause peripheral vasodilation leading to reduced cardiac preload
191. Inotropic activity oh the heart in CHF can be increased by which of the following agents?
I. Bipyridines
II. ACE inhibitors
III. Beta-1 agonists
a. I only
b. II only
c. I and III
d. II and III
e. I, II, and III
192. Which of the following agents for the heart failure may be associated with arrythmia,
thrombocytopenia, aand hepatotoxicity which limit their use in patients with chronic CHF?
A. Bipyridines
B. Digitalis glycosides
C. Beta-1 agonists
D. ACE inhibitors
E. Angiotensin II antagonists

Downloaded by ;%'%SiLeNt'%; (jemribsagpatan123@gmail.com)


lOMoARcPSD|5190755

193. Which of the following statements correctly describe/s the appropriate use of drugs for
CHF
I. Beta-blockers when used for patients with stable CHF can be given at doses similar to
the dose used for hypertension
II. ACE inhibitor dose must be carefully titrated to the maximum allowable or tolerable dose
III. Spironolactone has been found to be most useful as initial drug therapy for CHF even
prior to the use of ACE inhibitors or casodilators
a. I only
b. II only
c. I and III
d. II and III
e. I, II, and III
194. Which of the following drugs or drug combinations can reduce both the cardiac preload
and afterload in CHF?
I. Hydralazine + Nitrovasodilators
II. Beta-blockers + Loop diuretics
III. Loop diuretics + Digitalis
a. I only
b. II only
c. I and III
d. II and III
e. I, II, and III
195. Which of the following interventions may be beneficial in the management of CHF?
I. Replacement of chronic digoxi dose with ACE inhibitors
II. Add-on therapt with Aldosterone antagonist on patients already on digoxin or ACE-
inhibitors
III. Use of calcium channel blockers in patient who cannot recieve beta-blockers
a. I only
b. II only
c. I and III
d. II and III
e. I, II, and III
196. Which of the following drugs is a sodium channel blockers that can prolong the duration
of the action potential and dissociates from the channel with intermediate kinetics?
a. Lidocaine
b. Flecainide
c. Quinidine
d. Bretylium
e. Diltiazem
197. Which of the following drugs is a sodium channel blockers which has no significant effect
on the duration of the action potential and dissociate from the channel with slow kinetics?
a. Lidocaine
b. Flecainide
c. Quinidine
d. Bretylium
e. Sotalol
198. Which of the following beta-blockers have been found to be most useful as anti-arrythmic
agent/s?
I. Esmolol

Downloaded by ;%'%SiLeNt'%; (jemribsagpatan123@gmail.com)


lOMoARcPSD|5190755

II. Metoprolol
III. Propanolol
a. I only
b. II only
c. I and III
d. II and III
e. I, II, and III
199. Which of the following anti-arrythmic agents has been associated with lupus-like side
effects?
a. Quinidine
b. Lidocaine
c. Propafenone
d. Amiodarone
e. Procainamide
200. What is the drug of choice for the management of sustained ventricular arrythmia
associated with acute myocardial infarction?
a. Procainamide
b. Lidocaine
c. Amiodarone
d. Verapamil
e. Adenosine
201. What is the preferred drug for the management of ventricular arrythmias associated with
digitalis intoxication?
a. Procainamide
b. Lidocaine
c. Amiodarone
d. Verapamil
e. Adenosine
202. What is the preferred drug for the management of acute episodes of supraventricular
tachycardia?
a. Quinidine
b. Verapamil
c. Amiodarone
d. Bretylium
e. Adenosine
203. What is the preferred drug for the management of chronic paroxysmal supreventricular
tachycardia?
a. Quinidine
b. Verapamil
c. Amiodarone
d. Bretylium
e. Adenosine
204. What is the primary mechnism of action of the drug Amiodarone?
a. Prolong the action potential with block of the sodium channels
b. Shorten the action potential with block of the sodium channels
c. Prolong the action potential with block of the potassium channels
d. Shorten the action potential with block of the potassium channels
e. Inhibit intrinsic sympathetic stimulation by preventing activation of the enzyme adenylyl
cyclase

Downloaded by ;%'%SiLeNt'%; (jemribsagpatan123@gmail.com)


lOMoARcPSD|5190755

205. What side effect/s may be expected with the use of Amiodarone?
I. Hypothyroidism or Htperthyroidism
II. Fatal pulmonary fibrosis
III. Symptomatic bradycardia and heart block particularly in patients with atrioventricular
nodal disease
a. I only
b. II only
c. I and III
d. II and III
e. I, II, and III
206. Measurement of the serum ALT prior to and during therapy of arrythmia is indicated
when which of the following drugs for arrythmia is/are used?
a. Propafenone
b. Moricizine
c. Amiodarone
d. Flecainide
e. Diltiazem
207. Which of the following calcium channel blockers is most useful in arrythmia therapy?
a. Verapamil
b. Diltiazem
c. Nifedipine
d. Lidocaine
e. Propafenone
208. Which of the following effects can be attributed to histamine binding to its H1 receptor?
I. Extravascular smooth muscle contraction
II. Vascular smooth muscle contraction
III. Gastric acid secretion
a. I only
b. II only
c. I and III
d. II and III
e. I, II, and III
209. The triple response of Lewis following intradermal injection of Histamine includes which
of the following?
I. Sensory nerve ending stimulation producing flare
II. Erythema caused by contraction of vascular smooth muscles
III. Endothelian cell contraction leading to exudation of fluids
a. I only
b. II only
c. I and III
d. II and III
e. I, II, and III
210. Which of the following effects may be expected as a direct consequence of blockade of
the H1 receptots
I. Sedation
II. Resolution of initial allergic response
III. Anti-nausea and Anti-emetic effect (anti-motion sickness)
a. I only
b. II only

Downloaded by ;%'%SiLeNt'%; (jemribsagpatan123@gmail.com)


lOMoARcPSD|5190755

c. I and III
d. II and III
e. I, II, and III
211. Dimenhydrinate and Diphenhydramine are H1 antihistamines that belong to what
chemical class of agents?
a. Alkylamines
b. Phenothiazine
c. Piperazine
d. Ethanolamine
e. Ethylaminediamine
212. Which of the following agents is used primarily for its sedating property as the active
ingredient of an OTC sleeping aid called Unisom?
a. Carbinoxamine
b. Doxylamine
c. Hydoxyzine
d. Meclizine
e. Tripelenamine
213. What class of H1-antihistamines include drugs that sre used as components of OTC
“cold” medications
a. Alkylamines
b. Phenothiazine
c. Piperazine
d. Ethanolamine
e. Ethylaminediamine
214. What is the active metabolite of hydroxyzine which is also available as an H1-
antihistamine without significant sedating side-effect?
a. Fexofenadine
b. Astemizole
c. Cetirizine
d. Loratidine
e. Terfenadine
215. Which of the following H1 anti-histamine chemical groups possess significant atropine-
like effect?
a. Alkylamines
b. Phenothiazine
c. Piperazine
d. Ethanolamine
e. Piperidines
216. Which of the following H1-antihistamines has significant blocking effects on 5-HT
receptors?
a. Promethazine
b. Cyproheptadine
c. Pyrilamine
d. Carbinoxamine
e. Cetirizine
217. Which of the following H1-antihistamines has significant anti-cholinergic effect to be
useful for the management of dytonia and pseudoparkinsonism associated with the use of
neuroleptic agents?
a. Diphenhydramine

Downloaded by ;%'%SiLeNt'%; (jemribsagpatan123@gmail.com)


lOMoARcPSD|5190755

b. Brompheniramine
c. Promethazine
d. Cyproheptadine
e. Cyclizine
218. Which of the following H1-antihistamines can block sodium channels in excitable
membranes bringing about local anesthetic effect?
a. Diphenhydramine
b. Meclizine
c. Doxylamine
d. Pyrilamine
e. Cyproheptadine
219. What is the primary mechanism of action of the drug Ondansetron and Granisetron?
a. Partiaal agonist effect at the 5-HT1A receptor
b. Antagonist effect at the 5-HT1B receptor
c. Full agonist effect at the 5-HT2A receptor
d. Antagonist effect at the 5-HT3 receptor
e. Partial agonist effect at the 5-HT4 receptor
220. What is the clinical use of the drugs Ondansetron and Granisetron?
a. Appetite suppressants
b. Anti-emetic
c. Treatment of acute migraine attack
d. Migraine prophylaxis
e. Motility enhancing agent
221. What is the clinical use of the drugs that primarily stimulate the 5-HT1B and 5-HT1D
receptors?
a. Appetite suppressants
b. Anti-emetic
c. Treatment of acute migraine attack
d. Migraine prophylaxis
e. Motility enhancing agent
222. Sumatriptan is contrindicated in which subset of patients?
a. Patients with broncospastic disease
b. Patients with acute attacks of migraine
c. Patients with ischemic heart disease
d. Patients with hypertension
e. Patients with Wolff-Parkinson-White syndrome
223. Which of the following statements is/are correct regarding Arachidonic acid?
I. It is 20-carbon known as eicosapentaenoic acid
II. It is mobilized from phospholipids by the enzyme phospholipase A2
III. It is released from phospholipids by the action of the enzymes phopholipase C and
Diglyceride lipase
a. I only
b. II only
c. I and III
d. II and III
e. I, II, and III
224. Which of the following eicosanoids is a product of the metabolism of Arachidonic acid by
the cyclooxygenase enzyme?
a. Leukotrienes

Downloaded by ;%'%SiLeNt'%; (jemribsagpatan123@gmail.com)


lOMoARcPSD|5190755

b. Isoprostanes
c. Epoxides
d. Lipoxins
e. Prostaglandins
225. Which of the following correctly differentiate/s COX-1 from COX-2 isozymes?
I. COX-1 is widely distributed and constitutively expressed
II. COX-1 is expressed upon stimulation by sytokines and growth factors
III. COX-2 is expressed by inflammatory and immune cells
a. I only
b. II only
c. I and III
d. II and III
e. I, II, and III
226. Epoprostenol, a drug that is being used in the management of primary pulmonary
hypertension is an analogue of which eicosanoid?
a. PGD2
b. PGI2
c. PGE1
d. PGE2
e. PGF2-alpha
227. Which of the following eicosanoids has an analogue that is primary indicated as
cytoprotectant in peptic ulcer disease?

a. PGD2
b. PGI2
c. PGE1
d. PGE2
e. PGF2-alpha
228. Which of the following effects is caused by Prostacylin?
a. Bronchoconstriction
b. Vasoldilation of the blood vessels
c. Inhibition of gastric acid secretion
d. Contraction of the uterus
e. Inhibition of renin secretion
229. What is the primary indication of the prostaglandin analogue Latanoprost?
a. Management of glaucoma
b. Smooth muscle relaxant in erectile dysfunction
c. Abortifacient
d. Inhibition of platelet aggregation
e. Vasoconstrictor in episodes of hypertension
230. Which of the following statement correctly describes the effects of glucocorticosteroids in
eicosanoid synthesis?
I. Dexamethasone inhibits the expression of the COX-2 gene
II. Glucocorticosteroids inhibit the synthesis of annexins leading to decreased activity of
Phospholipase C
III. Glucocorticosteroids stimulates the syntheis of lipocortins which inhibit the activity of
Phospholipase A2
a. I only
b. II only

Downloaded by ;%'%SiLeNt'%; (jemribsagpatan123@gmail.com)


lOMoARcPSD|5190755

c. I and III
d. II and III
e. I, II, and III
231. Which of the following drugs classified as NSAIDs inhibit/s the COX-1 more than the
COX-2 isozymes?
a. Indomethacin
b. Ibuprofen
c. Mefenamic acid
d. Naproxen
e. Celecoxib
232. Which of the following drugs classified as NSAIDs inhibit/s the COX-2 morre than the
COX-1 isozymes?
a. Indomethacin
b. Ibuprofen
c. Mefenamic acid
d. Naproxen
e. Celecoxib
233. Which of the following statement/s is correctly describe/s the pharmacokinetics of
Aspirin?
I. Aspirin can be absorbed in the stomach and the small intestines in the hydrolyzed form
II. Elimination of aspirin can be best descrined as following a Michaelis-Menten kinetics
III. Acidification of the urine facilitates renal elimination of Salicylates
a. I only
b. II only
c. I and III
d. II and III
e. I, II, and III
234. Which of the following mechanisms may play a role of anti-inflammatory effect of Aspirin?
I. Decreased synthesis of prostaglandins
II. Inhibition of neutrophil migration and adherence
III. Stabilize iysosomes of neutrophils
a. I only
b. II only
c. I and III
d. II and III
e. I, II, and III
235. At what usual adult dose of aspirin is its anti-inflammatory effect observed?
a. 0.3-2.4g/day
b. <0.6-4g/day
c. 3.2-4g/day
d. <0.325g/day
e. At all dose range
236. What is the principal mechanism which can explain anti-pyretic effect of Aspirin?
a. Peripheral reduction in the levels of prostaglandins
b. Peripheral vasodilation
c. Inhibition of prostaglandin syntheis at subcortical sites
d. Inhibtion of CNS response to interleukin-1
e. Activation of the thermoregulatory sweat glands

Downloaded by ;%'%SiLeNt'%; (jemribsagpatan123@gmail.com)


lOMoARcPSD|5190755

237. Which of the following statements is/are correct regarding the anti-platelet aggregation
effect of Aspirin?
I. Discontinuation of Aspirin immediatel rrestores platelet aggregation in matter of 1-2 days
II. The mechanism involves irreversible actylation of the cyclooxygenase enzyme in
platelets
III. The effect is clinically significant in all dose ranges for Aspirin
a. I only
b. II only
c. I and III
d. II and III
e. I, II, and III
238. Which of the following statements must be considered with regard to the effect of Aspirin
aand other NSAIDs on the gastric mucosa?
I. The risk of GI intolerance can be reduced if Aspirin or any other NSAIDs is taken with
meals
II. With Aspirin, the risk of erosive gastritis is obsereved only when Aspirin is given at doses
>0.325g/day
III. Gastric irritation is most prominent among NSAIDs that inhibit COX-1 more than COX-2
a. I only
b. II only
c. I and III
d. II and III
e. I, II, and III
239. Which of the following drug is currently considered as the preferred drug to prevent
erosive gastritis in patient at risk of developing this side effect when NSAIDs have to be given to
the patient?
a. Omeprazole
b. Ranitidine
c. Sucralfate
d. Telenzepine
e. Misoprostol
240. In salicylate poisoning, serum salicylate levels of 50-80mg/dL can present with which of
the following clinical findings?
I. Hyperthermia
II. Respiratpry alkalosis
III. Tinnitus and Vertigo
a. I only
b. II only
c. I and III
d. II and III
e. I, II, and III
241. A25-years old male ingested an unidentified number of Aspirin tablets. He presents with
the following findings: Responsive to verbal stimulation, BP=110/60, Body Temperature=39.5’C,
ABGs: pH=6.9, HCO3=7meq/L, pCO2 -29 mmHg; dry oral mucosa, absent axillary sweating.
What serum salicylate level is consistent with these findings?
a. 50-80mg/dL
b. 80-110mg/dL
c. 110-160mg/dL
d. 160-190mg/dL

Downloaded by ;%'%SiLeNt'%; (jemribsagpatan123@gmail.com)


lOMoARcPSD|5190755

e. >190mg/dL
242. A number of the NSAIDs cannot be given to patients with Gouty Arthritis as these may
precipitate an acute attack of gout or induce uric acid stone formation. Which of the following
NSAIDs should no be given to patient with gout?
a. Tolmetin
b. Ibuprofen
c. Indomethacin
d. Diclofenac
e. Mefenamic acid
243. Which of the following NSAIDs with minimal anti-inflammatory activity is/are primarily
indicated as analgesic especially in the management of post-operative pain?
I. Diclofenac
II. Ketorolac
III. Etodolac
a. I only
b. II only
c. I and III
d. II and III
e. I, II, and III
244. All the know NSAIDs are weak acids. Which of the following drugs is the only exception?
a. Piroxicam
b. Sulindac
c. Nabumetone
d. Ketorolac
e. Tolmetin
245. Which of the following statement/s is/are correct regarding Mefenamic Acid, a popular
NSAID?
I. It is more toxic than Aspirin
II. The drug should not be used for more than 1 week
III. It should not be given to children
a. I only
b. II only
c. I and III
d. II and III
e. I, II, and III
246. What is the most important toxicity associated with pyrazolone derivatives like
Phenylbutazone which necessitated the withdrawal of a number of these drugs from the market?
a. Hepatoxicity
b. Hematologic toxicities
c. Nephrotoxicity
d. Gastric ulceration
e. Stevens-johnson syndrome
247. Which of the following NSAIDs is most useful as an analgesic and has been used
successfully to replace morphine in some situation involving mild to moderate post surgical pain?
When used with an opioid, it may decrease the opioid requirement by as much as 25-50%.
a. Naproxen
b. Ketorolac
c. Tenoxicam
d. Diclofenac

Downloaded by ;%'%SiLeNt'%; (jemribsagpatan123@gmail.com)


lOMoARcPSD|5190755

e. Mefenamic acid
248. What is the primary advantage od drugs like Celecoxib and etericoxib?
a. Less incidence of gastric irritation or ulceration
b. Less incidence of nephrotoxicity
c. Less incidence of hematologic toxicities
d. Minimal risk of causing worsening hypertension
e. Less likelihood of inhibiting platelet aggregation
249. A number of drugs classified as specificCOX-2 inhibitors like Rofecoxib (Vioxx) have
been withdrawn from the market or are marketed with “black box” warning. What is the primary
reason for such actions to be taken against these drugs?
a. They are associated with severe agranulocytosis
b. They cause severe irreversible hepatic necrosis
c. They increase the risk of bleeding in patient on Warfarin
d. They increase the risk of thrombosis and cardiac deaths
e. They cause worsening of DM and Hypertension
250. Which of the following agents is currently used as the first-line DMARDs in the
management of Rheumatoid Arthritis?
a. Hydroxychloroquine
b. Auranofin
c. Penicillamine
d. Azathioprine
e. Methotrexate
251. When the methotrexate is used for the management of rheumatoid arthitis, what is the
usual maintenance dosing regimen given to patients?
a. 150mg OD
b. 15mg OD
c. 15mg once a week
d. 150mg BID
e. 15mg BID
252. Which of the following statement/s is/are correct regarding the drug Methotrexate when
used for rheumatoid arthritis?
I. The primary mechanism of action involves inhibition of AICAR
(aminomidazolecarboxamide) transformylase and thymydylate synthase
II. Toxicity of the drug may be minimized with the use of Leucovorin
III. The drug is contraindicated in pregnancy
a. I only
b. II only
c. I and III
d. II and III
e. I, II, and III
253. Which of the following statements is/are correct regarding the drug acetaminophen?
I. It is a weak inhibitor of COX-1 and COC-2 isozymes in peripheral tissues
II. It has significant anti-inflammatory activity
III. It is equivalent to Aspirin as analgesic and anti-pyretic
a. I only
b. II only
c. I and III
d. II and III
e. I, II, and III

Downloaded by ;%'%SiLeNt'%; (jemribsagpatan123@gmail.com)


lOMoARcPSD|5190755

254. Which of the following agent is most useful in the management of acetaminophen
poisoning?
a. N-acetylcysteine
b. S-carboxymethylcysteine
c. Dantrolene
d. Atropine
e. Sodium Bicarbonate
255. Which of the following condition when present may increase of formation of the toxic
intermediate N-acetyl-p-iminobenzoquinone with regular doses of acteminophen?
I. Patient with viral hepatitis
II. Patients on warfarin therapy
III. Chronic alcoholics
a. I only
b. II only
c. I and III
d. II and III
e. I, II, and III
256. Which of the following drugs used for gout produces its effect by inhibiting the enzyme
xanthine oxide
a. Allopurinol
b. Colchicine
c. Indomethacin
d. Probenecid
e. Sulfinpyrazone
257. Which of the following drugs used for gout produces its effect by inhibiting the synthesis
of microtubules?
a. Allopurinol
b. Colchicine
c. Indomethacin
d. Probenecid
e. Sulfinpyrazone
258. Which of the following is/are appropriate indication/s of Allopurinol?
I. In the management of chronic tophaceous gout
II. In preventing massive uricosuria in patients to be given chemotherapeutic agents for
acute leukemia
III. In patients with recurrent renal urate stones
a. I only
b. II only
c. I and III
d. II and III
e. I, II, and III
259. In the management of acute gouty attack, which of the following statement/s is/are
correct?
I. Uricosuric agent are given at the onset of acute attack to reduce the duration of the
episode
II. Indomethacin is preffered for the management of the pain of an acute attack
III. Colchine relieves the pain and inflammation of an acute attack in 12-24 hours
a. I only
b. II only

Downloaded by ;%'%SiLeNt'%; (jemribsagpatan123@gmail.com)


lOMoARcPSD|5190755

c. I and III
d. II and III
e. I, II, and III
260. Which of the following groups can relax the brochial smooth muscles and are primarily
used as bronchodilators in bronchial asthma?
I. Methylxanthines
II. Beta-2 agonists
III. Anticholinergic agents
a. I only
b. II only
c. I and III
d. II and III
e. I, II, and III
261. In the management of bronchial asthma, which of the following medications
is/areconsidered as controller medications?
I. Low-dose inhaled glucocorticosteroids
II. Short-acting beta-2 agonists
III. Long-acting inhaled beta-2 agonists
a. I only
b. II only
c. I and III
d. II and III
e. I, II, and III
262. Which of the following advice should you give to a patient with bronchial asthma
regarding the use of medications for his condition based on the current recommendations?
I. Oral bambuterol must be taken on PRN basis for acute exacerbations of bronchial
asthma
II. Inhalation of salbutamol for acute relief of attack mest be used for not more than three
times a day
III. He must adequately gargle about 5x after each use Fluticasone inhaler
a. I only
b. II only
c. I and III
d. II and III
e. I, II, and III
263. Which of the following drugs for bronchial asthma can be given subcutaneously during
acute episodes of bronchospasm?
a. Salnutamol
b. Isoproterenol
c. Terbutaline
d. Budesonide
e. Theophylline
264. Which of the following mechanisms may explain the actions of methylxanthines in
bronchial asthma?
I. May inhibit the late phase allergic reaction occuring about 2-8hours after an acute attack
II. May inhibit the enzyme phosphodiesterase leading to an increase in the levels of cAMP
III. May antagonize the action of Adenosine
a. I only
b. II only

Downloaded by ;%'%SiLeNt'%; (jemribsagpatan123@gmail.com)


lOMoARcPSD|5190755

c. I and III
d. II and III
e. I, II, and III
265. Which of the following statements describe/s the clinical use of Theophylline?
I. May reduce the nocturnal attack of bronchial asthma when given as sustained-release
preparation
II. May improve response to high dose inhaledd beta-2 agonists when given intravenously
concurrently with beta-2 agonists during acute exacerbation of asthma
III. May improve contratility of the diaphragm improving ventilatory response to hypoxia
a. I only
b. II only
c. I and III
d. II and III
e. I, II, and III
266. Which of the following statements is an important consideration regarding the toxicities
and side effects of Theophylline?
a. Toxicities and side effects do not occur as long as the serum levels of Theophylline do
not exceed 20mg/L
b. Adults, neonates, and young infants have faster clearance of Theophylline than children
c. Clearance may be decreased with inhibition of hepatic enzymes among chronic smokers
d. Seizures and arrythmias are more common when serum levels exceed 40mg/L
e. Hepatic dysfunction does not alter significantly the clearance of Theophylline and can
thus be safely used for patients with liver disease
267. Which of the following statements describe the use of anti-cholinergic agents in COPD
and bronchial asthma?
I. They can be safely given at high doses by inhalations to saturate the muscarinic
receptors
II. Available agents are tertiary ammonium compunds
III. They are more effective than beta-2 agonists in bronchial asthma but less effective in
COPD
a. I only
b. II only
c. I and III
d. II and III
e. I, II, and III
268. What is the mechanism of action of Nedocromil?
a. Increase outward potassium conductance leading to hyperpolarization of mast cell
membrane
b. Increase inward chloride conductance leading to hyperpolarization of mast cell
membrane
c. Increase inward sodium conductance leading to depolarization of mast cell membrane
d. Increase outward calcium conductance leading to repolarization of mast cell membrane
e. Increase inward potassium conductaance leading to depolarization of mast cell
membrane
269. What is the mechanism of action of Zafirlukast and Montelukast?
a. Pharmacologic antagonism of Luekotriene D4 action
b. Pharmacologic antagonism of Leukotriene C4 action
c. Inhibition of the enzyme 5-lipoxygenase
d. Inhibition of citokine release by macrophages

Downloaded by ;%'%SiLeNt'%; (jemribsagpatan123@gmail.com)


lOMoARcPSD|5190755

e. Inhibition of Histamine-1 receptors


270. Which of the following agents has/have experimental effect as bronchodilator/s?
I. Omalizumab
II. Nifedipine
III. Nitric oxide
a. I only
b. II only
c. I and III
d. II and III
e. I, II, and III
271. Which of the following correctly describes the action of cromolyn sodium?
a. Induces relaxation of bronchial smooth muscles
b. Blocks the degranulation of mast cells
c. Inhibits release of cytokines by inflammatory cells
d. Inhibits binding of IgE to mast cell membrane
e. Inhibits degradation of cGMP
272. Which of the following effects observe with acute or chronic use of opioids like morphine
can be attributed to stimulatio of the mu-receptors?
I. Respiratory depression
II. Psychosis
III. Miosis
a. I only
b. II only
c. I and III
d. II and III
e. I, II, and III
273. Which of the following statement correctly describe the mechanism of action of the
opioids?
I. They mimic the action of endogenous opioid peptides
II. The stimulate the release of endogenous opioid peptides
III. They bind to receptors like the mu, kappa, and delta
a. I only
b. II only
c. I and III
d. II and III
e. I, II, and III
274. Which of the following is an opioid receptor that is responsible for inducing analgesia
most especially among women?
a. Kappa
b. Delta
c. Alpha
d. Epsilon
e. Mu
275. Tolerance is an expected consequence with chronic use of most of the available opioids.
Which of the following effects of opioids is tolerance NOT observed?
a. Respiratory depression
b. Analgesia
c. Euphoria
d. Constipation

Downloaded by ;%'%SiLeNt'%; (jemribsagpatan123@gmail.com)


lOMoARcPSD|5190755

e. Sedation
276. Which of the following statements regarding the development of tolerance to opioids
is/are correct?
I. Develops in 2-3 weeks of continuous use especially when large doses are used over short
intervals
II. Gross-tolerance develops among predominantly kappa-receptor full agonists
III. Partial agonists induce cross-tolerance with strong agonists and antagonists
a. I only
b. II only
c. I and III
d. II and III
e. I, II, and III
277. A 45-year old male admitted for severe chest painwas diagnosed with AMI. He was
being given morphine 2mg IV for relief of the chest pain. Which of the following is/are important
findings that must be observed as expected consequence/s of the use of Morphine?
I. Tachycardia
II. Hypotension
III. Vasospasm
a. I only
b. II only
c. I and III
d. II and III
e. I, II, and III
278. Which of the following opioids is not associated with biliary colic and thus can be used
for severe pain associated with this condition?
a. Morphine
b. Pentazocine
c. Fentanyl
d. Meperidine
e. Codeine
279. Abstinence syndrome is a rebound effect characterized by rhinorrhea, lacrimation,
yawning, chills, gooseflesh, hyperventilation, and hostility observed in which of the following
situations?
I. Abrupt withdrawal of an opioid agonist in a patient taking the drug chronically
II. In a newborn of a mother illicitly taking heroin during her pregnancy
III. Administration of Nalbuphine on a patient chronically on morphine
a. I only
b. II only
c. I and III
d. II and III
e. I, II, and III
280. Which of the following oral doses of morphine is quivalent to 10mg of an IV dose?
a. 200mg
b. 180mg
c. 100mg
d. 40mg
e. 10mg
281. Diacetylmorphine is the chemical name of which of the following opioids?
a. Apomorphine

Downloaded by ;%'%SiLeNt'%; (jemribsagpatan123@gmail.com)


lOMoARcPSD|5190755

b. Morphine
c. Heroin
d. Codeine
e. Meperidine
282. Tramadol is weak mu-receptor agonist that does not require as S2 license for
dispensing. It is a synthetic analogue ofwhich of the following opiates/opioids?
a. Codeine
b. Thebaine
c. Morphine
d. Meperidine
e. Diphenoxylate
283. Which of the listed opioid below is along the recommended drugs for the management of
acute pulmonary edema?
a. Nalbuphine
b. Codeine
c. Morphine
d. Meperidine
e. Apomorphine
284. What is the primary mechanism involved in the utility of opioids in the management of
acute pulmonary edema?
a. Induction of osmotic diiuresis of edema fluuids
b. Peripheral vasodilation reducing venous return
c. Decreased cardiac inotropism
d. Improved mobilization of edema fluid by osmosis
e. Increased glumerolar filtration rate
285. Which of the following is/are possible therapeutic classification/s for available opiates
and opioids?
I. Cough supressants
II. Emetics
III. Analgesics
a. I only
b. II only
c. I and III
d. II and III
e. I, II, and III
286. What is the main reason why atropine is always given with diphenoxylate (lomotil)?
a. The anticholinergic after-effects with Atropine discourages abuse of Diphenoxylate
b. The anti-motility effect of Atroppine augments the constipating effect of Diphenoxylate
c. Atropine minimizes the respiratory depression expected with Diphenoxylate
d. Atropine reduces the absorption and systemic toxicity of Diphenoxylate
e. Atropine inhibits the hepatic metabolism of diphenoxylate effectively reducing the dose
requuired for diarrhea
287. What is the primary indication for the drug methadone?
a. Analgesic substitute for Morphine
b. Antitussive substitute for Codeine
c. Weaning off patients addicted to Heroin or Morphine
d. Management of biliary colic
e. Anesthetic premedication
288. Which of the following agents is a strong full agonist of opioid receptors

Downloaded by ;%'%SiLeNt'%; (jemribsagpatan123@gmail.com)


lOMoARcPSD|5190755

a. Hydromorphone
b. Hydrocodone
c. Codeine
d. Propoxyphene
e. Pentazocine
289. Which of the following agents is classified as a partial agonist of opioid receptors?
a. Meperidine
b. Methadone
c. Levallorphan
d. Fentanyl
e. Nalbuphine
290. Which of the following agents is/are classified as opioid antagonist/s?
I. Nalorphine
II. Naloxone
III. Naltrexone
a. Meperidine
b. Methadone
c. Levallorphan
d. Fentanyl
e. Nalbuphine
291. Which of the following anticoagulant is produced by recombinant DNA technology from
the gene that encodes the natural anticoagulant derived from medicinal leeches?
a. Dicumarol
b. Hirudin
c. Lepirudin
d. Heparin
e. Dalteparin
292. What is the primary indication of the anticoagulant derived from medicinal leeches or its
recombinant form?
a. Anticoagulation when necessary during pregnancy
b. When initiatin anticoagulation therapy in patient requiring chronic anticoagulation
c. Treatment of and prophylaxis against pulmonary thromboembolism
d. Management of thrombosis associated with heparin-induced thrombocytopenia
e. In patients with AMI or unstable angina pectoris
293. What is the mechanism of action of regular heparin?
a. Activates antithrombin III which in turn inactivates a number of activated clotting factors
b. Activation of plasminogen to plasmin leading to destruction of fibrin and fibrinogen
c. Inhibits the reductive activation of vitamin K epoxide to the hydroquinone form
d. Inhibits activvation of prothrombin to thrombin
e. Mimics the action of epsilon-aminocaproic acid
294. The activity of regular heparin when given intravenously is best monitored atleast every
6 hours for the first 24 hours of therapy using what parameter?
a. Protime
b. Platelet count
c. Clot reaction time
d. Activated partial thromboplastin time
e. Bleeding time
295. When regular heparin is used subcutaneously, which of the followinh statements is
correct?

Downloaded by ;%'%SiLeNt'%; (jemribsagpatan123@gmail.com)


lOMoARcPSD|5190755

a. There is no need to monitor activity in most cases


b. Monitoring of activity must be done on a weekly instead of daily basis
c. Monitoring must be done using aPTT
d. Monitoring of activity must be done regularly using the PT value
e. Monitoring of activity must be done using the platelet count
296. Which of the following agents must be classified as a low molecular weight heparin?
a. Enoxaparin
b. Regular heparin
c. Warfarin
d. Hirudin
e. Phenindione
297. Which of the following parameters must be measured when a patient on low molecular
weight heparin develops bleeding that may be associated with therapy?
a. aPTT
b. Prothrombin time
c. Factor X assay
d. Factor IX assay
e. Bleeding time
298. Bleeding associated with the use of regular heparin is best managed with which of the
following interventions?
a. Immediate stopping of therapy
b. Administration of Tranexamic acid
c. Administration of vitamin K
d. Administration of protamine sulfate and stopping therapy
e. Lowering of the current dose without stopping therapy
299. What is the primary drawback with phenprocoumon that limit its clinical use in the
management of thrombosis?
a. Ist metabolism to a hepatotoxic metabolite
b. Ita long half-life
c. Its prominent gastrointestinal side effects
d. Its high degree of protein binding
e. The long delay in its onset of action
300. Warfarin exerts its effect primarily by inhibiting the gamma-carboxylation of the
glutamate residues of which of the following clotting factors?
a. IXa,Xa, XIa, XIIIa
b. IIa, VIIa, IXa, Xa
c. II, VII, IX, X
d. IX, X, XI, XIII
e. I, II, V, VII
301. When warfarin is used as anticoagulant, the achievement of a maximal
hypothrombinemic effect is expected to be delayed. What is the primary reason for this delay in
warfarin effect?
a. The presence of preformed clotting factors in the plasma which have biiologic half-lives 6
to 60 hours
b. The high degree of protein binding which dela entry of the drug into its biologic site of
action
c. The delay of activation of precursor clotting factors due to high levels of activation-
inhibitors

Downloaded by ;%'%SiLeNt'%; (jemribsagpatan123@gmail.com)


lOMoARcPSD|5190755

d. The ability of warfarin to induce its own metabolism which delays achievement of
therapeutic levels by days
e. The ability of warfarin to interfere with the assay done to measure its activity
302. Cutaneous necrosis which can develop during the frist few weeks of therapy is expected
with warfarin due to which of the following mechanisms?
a. Idiosyncratic and paradoxical increase in the rate of synthesis of a number of clotting
factors
b. Earlier decline in the circulating levels of natural anticoagulants causing predominance of
procoagulant clotting factors
c. Intrinsic thrombogenic property of warfarin molecules
d. Direct reduction in tissue perfusion as a consequence of the allergic property of warfarin
e. Hypotensive effective of warfarin which reditributes blood flow to critical sites in the body
303. What is the recommended target protime INR for patients who need warfarin and have
prosthetic heart valves?
a. 1-3
b. 2-3
c. 3-4
d. 4-5
e. 5-6
304. A patient on warfarin for the last 4 weeks for DVT has a PT INR of 4 but without
evidence of bleeding. What is the most appropriate intervention?
a. No intervention as within target INR range
b. Reduction of the dose by atleast half
c. Adimintration of vitamin K
d. Adminitration of protamine sulfate
e. Administration of tranexamic acid
305. Which of the following drugs or conditions can increase the activity of warfarin by
affecting the drug’s absorption, distribution, and elimination?
a. Cimetidine
b. Rifampicin
change to vegetarian diet
c. The presence of hepatic disease
d. Concurrent use of heparin
306. What interaction is present with concominant use of warfarin and the pyrazolone
NSAIDs?
I. Inhibition of urinary excretion of warfarin
II. Inhibition of the oxidative metabolism of S-Warfarin
III. Displacement of Warfarin from its protein binding
a. I only
b. II only
c. I and III
d. II and III
e. I, II, and III
307. Which of the following correctly describes the mechanism of action of fibrinolytic agents?
a. Directly catalyze the proteolysis of fobrin and fibrinogen
b. Inhibit the action of thrombin by preventing its binding to fibrin and fibrinogen
c. Activate plasminogen to plasmin, a serine protease that catalyzes breakdown of fibrin
and fibrinogen
d. Inhibit activation of fibrinogen by proteolysis of thrombin

Downloaded by ;%'%SiLeNt'%; (jemribsagpatan123@gmail.com)


lOMoARcPSD|5190755

e. Activate the endogenous anti coagulants protein C and protein S


308. Which of the following is classified as a thrombolytic agent?
a. Reteplase
b. Warfarin
c. Argatroban
d. Bivalirudin
e. Lepirudin

309. Which of the following statements is/are correct regarding the use of thrombolytic agents in
myocardial infarction?

I. Best response is seen if the drug is given within 24 hours from onset of symptoms

II. Generally given to patients with ST segment elevation

III. Absolute contraindication is a history of stroke of less than 1 year

a. I only d. II and III


b. II only e. I, II and III
c. I and III

310. Which of the following agents is associated with formation of antibodies that can cause development
of allergic reaction with subsequent exposure to the drug?

a. Urokinase d. Tissue plasminogen Activator

b. Streptokinase e. Tenecteplase

c. Alteplase

311. Which of the following mechanisms may lead to inhibition of platelet aggregation?

I. Activation of antithrombin III

II. Thromboxane synthesis inhibition

III. Guanylyl cyclase inhibition

A. I only d. II and III


B. II only e. I, II and III
C. I and II

312. Which of the following mechanisms correctly describes the mechanism of action of aspirin as an
inhibitor of platelet aggregation?

A. Irreversible inhibition of platelet and endothelial cell cyclooxygenase

B. Irreversible inhibition of endothelial cell cyclooxygenase

C. Irreversible inhibition of platelet cyclooxygenase

D. Reversible inhibition of platelet and endothelial cell cyclooxygenase

E. Reversible inhibition of endothelial cell cyclooxygenase

313. Which of the following agents requires weekly WBC monitoring within the first 3 months of treatment
due to a risk of causing leukopenia?

Downloaded by ;%'%SiLeNt'%; (jemribsagpatan123@gmail.com)


lOMoARcPSD|5190755

A. Cilostazol C. abciximab

B. Dipyridamole D. Clopidogrel

E.Ticlodipine

314. Which of the following agents is associated with development of thrombotic thrombocytopenic
purpura?

A. Aspirin D. Eptifibatide

B. Ticlodipine E. Cilostazol

C. Tirobifan

315. Which of the following agents is classified as an inhibitor of glycoprotein IIb/IIIa and is useful in
patients with acute coronary syndrome and those undergoing percutaneous coronary intervention?

A. Clopidogrel D. Cilostazol

B. Ticlodipine E. Urokinase

C. Abciximab

316. What is the most important adverse effect of Dipyridamole when given to patients with ischemic
heart disease?

A. Significant thrombocytopenia D. Severe diarrhea

B. Vasospasm of coronary vessels e. intermittent claudication

C. Coronary steal phenomenon

317. What is the mechanism of action of the drug Tranexamic acid?

A. ADP inhibition D. Inhibits reductase hydrolysis of Vitamin K epoxide

B. Serine protease inhibition E. Increase factor VIII activity

C. Inhibits plasminogen activation

318. Which of the following agents primarily cause lowering of the VLDL and is thus useful for
hypertiglyceridemia?

A. Niacinamide D. Probucol

B. Fenofibrate E. Cholestyramine

C. Simvastatin

319. Which of the following is consistent with the action of the drug Colestipol?

I. Prevents reabsorption of bile acids in intestine

II. Increases uptake of LDL and IDL with up-regulation of LDL receptors in the liver

III. Causes an increase in conversion of cholesterol to bile acids

A. I only D. II and III

Downloaded by ;%'%SiLeNt'%; (jemribsagpatan123@gmail.com)


lOMoARcPSD|5190755

B. II only E. I, II and III


C. I and III

320. What is the mechanism of action of Atorvastatin?

A. Inhibits the first committed step in the biosynthesis of cholesterol

B. Serves as ligand for the nuclear transcription called peroxisome proliferators-activated receptor alpha

C. Stimulates the activity of lipoprotein lipase leading to lipolysis of lipoprotein triglyceride

D. Inhibits the reuptake of bile acids leading to loss of negative feedback on the alpha-hydroxylase
enzyme

E. Prevents oxidation of LDL cholesterol

321. Which of the following is/are important monitoring considerations in a patient receivimg Simvastatin?

I. Baseline and follow-up measurement of serum creatinine

II.Baseline and follow-up measurement of amino transferase activity

III. Baseline and follow-up measurement of creatine kinase activity

A. I only D. II and III


B. II only E. I, II and III
C. I and III

322. Patients on Statins are generally advised to take the medication at night. What is the basis for this?

A. Maximal absorption occurs at night D. Minimize Syncopal attacks with the first dose of statins

B. Maximal cholesterol de-novo synthesis occurs at nightE. LDL release occurs at night

C. Minimize postural hypotension expected with Statins

333. Statins are generally best taken at night. Which of the following statins is night dosing unnecessary?

A. Lovastatin D. Pravastatin

B. Simvastatin E. Fluvastatin

C. Atorvastatin

324. Which of the following drug combinations is generally avoided?

A. Simvastatin + Niacin D. Gemfibrozil + Colestipol

B. Niacin + colestipol E. Simvastatin + Colestipol

C. Simvastatin + Gemfibrozil

325. Therapy with HMGCoA reductase inhibitors is associated with Rhabdomyolysis and myositis. These
complications which can lead to acute tubular necrosis and renal failure have been reported to be
augmented with co-administration of which drugs.

A. Fenofibrates and other fibrates D. Tranylcypromine and other MAOI

B. Cholestyramine and other

Downloaded by ;%'%SiLeNt'%; (jemribsagpatan123@gmail.com)


lOMoARcPSD|5190755

bile acid binding resins E. Methampethamine and other Norepinephrine releasers

D. Captopril and other ACE inhibitors

326. Which of the following drugs is associated with cutaneous vasodilation and warmth sensation after
an initial dose which may be reduced with the use of 300mg dose Aspirin taken an hour before the dose?

A. Niacin D. Cholestyramine

B. Fenofibrate E. Probucol

C. Atorvastatin

327. Which of the following drugs can be bind to molecular components of the macromolecular complex
of GABA-A receptor –chloride channel present in neuronal membranes in the CNS?

I. Diazepam

II. Zolpidem

III. Phenobarbital

A. I only D. II and III


B. II only E. I, II and III
C. I and III

328. Which of the following statements correctly describes the mechanism of action of the anxiolytic drug
Buspirone?

A. Selective binding to the BZ-1 receptor in the brain D. Inhibitor of MAO enzyme

B. Antagonist of alpha receptors in the brain E. Antagonist of the BZ-1 receptor in the brain

C. Partial agonist of 5HT-1A receptor in the CNS

329. Which of the following benzodiazepines is a prodrug that is hydrolyzed to the active metabolite
Nordiazepam in the stomach?

A. Alprazolam D. Diazepam

B. Chlordiazepoxide E. Lorazepam

C. Chlorazepate

330. What is the primary use of the drug Flumazenil?

A. Antagonizes the action of the Benzodiazepines

B. Stimulates BZ-1 receptor similar to benzodiazepines.

C. Antagonizes the action of Barbiturates

D. Inhibits the 5HT-1A receptor

E. Stimulates the GABA-A receptor

331. Which of the following Benzodiazepines has the shortest half-life and a rapid onset of action?

Downloaded by ;%'%SiLeNt'%; (jemribsagpatan123@gmail.com)


lOMoARcPSD|5190755

A. Lorazepam

B. Triazolam

C. Oxazepam

D. Diazepam

E. Prazepam

332. Which of the following agents is absolutely contraindicated in patients with disorder associated with
porphyrin such as acute intermittent porphyria, vanegate porphyria, and symptomatic porphyria?

A. Chlordiazepoxide

B. Diazepam

C. Zolpodem

D. Phenobarbital

E. Buspirone

333. Which of the following agents is associated with significant anterogade amnestic effect which is
advantageous in a number of medical and surgical procedures that may discomfort to the patient?

A. Midazolam

B. Zolpidem

C. Buspirone

D. Phenobarbital

E. Thiopental

334. What is the rationale behind the use of long-acting benzodiazepine like chlordiazepoxide and
diazepam in the management of alcohol withdrawal?

a. The benzodiazepines produce a paradoxical reaction among chronic alcoholics

B. Cross-dependence develops among sedative hypnotics

C. Increase in renal clearance of alcohol

D. Increase in non-CYP mediated metabolism of alcohol

335. Which of the following agents may be expected to have an additive CNS depressants effect with the
sedative hypnotics like benzodiazepines when taken simultaneously?

I.Opioid analgesics

II. Phenothiazines

III. Alcoholic beverages

A. I only
B. II only
C. I and III

Downloaded by ;%'%SiLeNt'%; (jemribsagpatan123@gmail.com)


lOMoARcPSD|5190755

D. II and III
E. I, II and III

336. Which of the following patients may be expected to have an increased sensitivity to sedative-
hypnotics leading to higher incidence of adverse effects or toxicity?

I. Young women

II. Patients with Cardiovascular disease

III. Patients with respiratory disease

A. I only
B. II only
C. I and III
D. II and III
E. I, II and III

337. Which of the following statements correctly describes the pharmacokinetics of Phenytoin?

A. Absorption of Phenytoin is constant regardless of the formulation

B. Phenytoin is well absorbed after intramuscular injection

C. Elimination of Phenytoin follows a first-order kinetics at very low blood levels

D. A first order kinetics of metabolism is observed with Phenytoin at low and high blood levels

E. Phenytoin is not significantly Protein bound so drug displacement interaction is not a major concern

338. Which of the following mechanisms explain/s the action of Phenytoin?

I. The drug alters conductance of sodium, potassium, and calcium

II. At high concentrations, it also inhibits the release of Serotonin and Norepinephrine

III. At therapeutic concentrations, Phenytoin inhibits the generation of repetitive action potentials

A. I only
B. II only
C. I and III
D. II and III
E. I, II and III

339. Which of the following findings in a patient taking Phenytoin would warrant reduction in the dose?

A. Gingival Hyperplasia

B. Nystagmus

C. Osteomalacia

D. Ataxia

E. Hirsutism

340. Autoinduction of the metabolism is a phenomenon that leads to decrease in the usual half-life of a
given drug that the dose adjustments must be made within the first month of therapy to ensure

Downloaded by ;%'%SiLeNt'%; (jemribsagpatan123@gmail.com)


lOMoARcPSD|5190755

therapeutic drug levels are reached, and reduction of dosage must be made once metabolism has
stabilized. Such is seen with which of the following drugs?

A. Carbamazepine

B. Phenytoin

C. Phenobarbital

D. Valproic acid

E. Primidone

341. Which of the following drugs for seizures is metabolized to yield Phenobarbital?

A. Lamotrigine

B. Vigabatrin

C. Primidone

D. Topimarate

E. Felbamate

342. When Phenytoin is given intravenously, what is the maximum rate by which drug can be
administered?

A. 10mg/min

B. 20mg/min

C. 30mg/min

D. 40mg/min

E. 50mg/min

343. Which of the following statements is/are correct regarding intravenous Phenytoin?
I. Cardiotoxicity is partly due to Propylene Glycol

II. It is compatible with Dextrose containing liquid

III. Fosphenytoin is more soluble and requires no solubilizing agent

A. I only
B. II only
C. I and III
D. II and III
E. I, II and III

344. Which of the following statements is/are correct regarding the toxicity profile and the management of
overdose of most drugs used for treatments of seizures?

I. The most dangerous effect is cardiotoxicity

II. Alkalinization of the urine to hasten elimination is usually effective

III. Stimulants are useful antidotes

Downloaded by ;%'%SiLeNt'%; (jemribsagpatan123@gmail.com)


lOMoARcPSD|5190755

A. I only
B. II only
C. I and III
D. II and III
E. I. II and III

345. What is the most common side effect seen with the use of carbamazepine which requires adjustment
in the dose of the drug?

A. Diplopia D. Vomiting

B. Drowsiness E. Leukopenia

C. Hyponatremia

346. What is the preferred drug for the management of pure absence seizures?

Downloaded by ;%'%SiLeNt'%; (jemribsagpatan123@gmail.com)


lOMoARcPSD|5190755

A. Phenobarbital

B. Valproic acid

C. Phenytoin

D. Ethosuximide

E. Lamotrigine

347. Which of the following agent is associated with the idiosyncratic hepatotoxicity when given for the treatment of seizure
in children less than 2 years of age and in patients with multiple medications?

A. Gabapentin

B. Valproic acid

C. Phenytoin

D. Primidone

E. Topimarate

348. Which of the following agents is currently the preferred drug for the initial management of Status Epilepticus?

A. Lorazepam

B. Diazepam

C. Phenobarbital

D. Phenytoin

E. Valproic acid

349. Which of the following drugs used or seizure disorders is most commonly associated with cosmetic changes making
the drug desirable to used among adolescents, school-age children, and women?

A. Lamotrigine

B. gabapentin

C. Phenobarbital

D. Phenytoin

E. Valproic acid

350. A 50 year old male with Generalized Tonic-clonic seizure was initially given Carbamazepine as maintenance therapy.
He however developed adverse response to drug in form of Stevens-Johnsons syndrome. Which of the following agents
may be safed to used as an alternative treatment for the seizure?

I. Valproic acid

II. Phenytoin

III. Phenobarbital

A. I only
B. II only
C. I and III
D. II and III
E. I, II and III

351. Which of the following drugs classified as diuretics maybe useful in epileptic women who have seizures exacerbations
at the time of menses?

A. Hydrochlorothiazide

B. Metolazone

C. Acetazolamide

D. Chlorthalidone

E. Torsemide

352. Which of the following correctly describes the clinical used of Carbidopa in the therapy of parkinsonism?

I. Increased the CNS bioavailability of Levodopa

II. Improve symptoms of bradykinesia

Downloaded by ;%'%SiLeNt'%; (jemribsagpatan123@gmail.com)


lOMoARcPSD|5190755

III. Minimize nausea and vomiting associated with Levodopa

A. I only
B. II only
C. I and III
D. II and III
E. I,II and III

353. What is the mechanism of action of Bromocriptine in the management of Parkinsonism?

A. Direct antagonist of D2 receptors

B. Direct agonist of D2 receptors

C. Direct antagonist of 5HT3 receptors

D. Direct agonist of 5HT3 receptors

E. Metabolized in the CNS to Dopamine

354. Female patients with Pituitary tumor and manifesting with amenorrhea and galactorrhea are initially manage with
Bromocriptine. What is the mechanism of Bromocriptine in this condition?

A. It inhibits the action Dopamine

B. It inhibits the action of Oxytocin

C. It stimulates the release of Estrogen

D. It inhibits the action of Prolactin

E. It stimulates the release of FSH and LH

355. During the maintenance therapy with Levodopa, patients are advsed to take the drug on an empty stomach. Why is
this advice given?

A. To minimize the risk of aspiration in as much as the drug can cause nausea and vomiting

B. Oral bioavailability of the drug is reduced as Levodopa can be found to food particles in the GIT

C. Bile acids released in response to food can interfere with the absorption of the drug

D. Large neutral amino acids in food interfere with the drugs’ transport acroos the blood brain barrier
E. Food can induce expression of DOPA decarboxylase which can limit the drugs CNS bioavailability

356. Which of the following drugs is most useful in controlling the bradykinesia of Parkinsonism?

A. Levodopa

B. Bromocriptine

c. Amantadine

D. Selegiline

E. Entacapone

357. Retroperitoneal fibrosis is an observed complication seen with drugs that belong to what class of agents?

A. Antiviral agents

B. Tropane Alkaloids

C. Ergot alkaloids

D. Ethanolamine Anti-histamines

E. Dopamine Agonist

358. Livedo Reticularis, a vascular cutaneous reaction characterized by a reddish-purple, fish net- patterm mottling of the
Upper/Lower extremities, is a side effect associated with which of the following agents?

A. amantadine

B. Di[henhydramine

C. Levodopa

D. Pramipexole

E. Selegiline

359. What is Entacapone?

Downloaded by ;%'%SiLeNt'%; (jemribsagpatan123@gmail.com)


lOMoARcPSD|5190755

A. A selective MAO-A inhibitor

B. A selective MAO-B inhibitor

C A COMT inhibitor

D. A Dopamine-3 receptor Agonist

E. A Dopamine-2 receptor agonist

360. The so-called “wearing-off” phenomenon encountered with the use of Levodopa-Carbidopa combination in
Parkinsonism is partly due to the formation of which metabolite which can compete with the transport of Levodopa across
the blood brain barrier?

A. alpha-methyldopamine

B. alpha-methylnorepinephrine

C. alpha-methyldopa

D. 3-O – methyldopa

E. 3-0- methyldopamine

361.Which of the currently available inhalation anesthetic is the most potent based in the minimum Alveolar concentration?

A. Halothane

B. Desflurane

C. Nitrous Oxide

D. Sevoflurane

E. Enflurane

362. Based on recent studies what is currently considered as the most probable explanation for the action inhalation
anesthetic?
A. Interaction with the molecular components of the GABA-A receptor

B. Interaction and interference with the activity of the fast inward sodium channels

C. Non-specific interaction with the lipid matrix of the nerve membrane

D. Inhibition of the interaction with Glutamate of the NMDA receptor

E. Inactivation of the Ligand-gated Potassium, channels

363. Which of the following inhalational anesthetics sensitizes the myocardium to cathecolamines which can lead to
ventricular arryhtmia when symphatomimetic drugs are co-currently administered?

A. Sevoflurane

B. Isoflurane

C. Desflurane]

D. Halothane

E. Nitrous Oxide

364. Which of the following inhalational anesthetics associated with potentially severe and Life-threatening hepatitis?
A. Sevoflurane

B Isoflurane

C. Desflurane

D. Halothane

E. Nitrous Oxide

365. Malignant Hyperthermia which may developed in susceptible individuals exposed to i9nhalational anesthetic is
appropriately treated with which of the following agents?

A. Diazepam

B. Baclofen

C. Dantrolene

D, Thiamylal

E. Haloperidol

Downloaded by ;%'%SiLeNt'%; (jemribsagpatan123@gmail.com)


lOMoARcPSD|5190755

366. Which of the following drugs combination provide a state called Neurolepanesthesia?

A. Haloperidol + Thiamylal + Halothane

B. Droperidol + Fentanyl + Nitrous Oxide

C. Thioridazine + succinyl choline + propofol

D. Nitroue Oxide + Droperidol +Fentanyl

E. Halothane + Drperidol + fentanyl

367. Which of the following statement is correct regarding the use of a combination of the least potent Nitrous oxide with a
second inhalation anesthetic?

I. Reduction in the risk of the myocardial depression at a given depth of anesthesia compared to the more potent
anesthetic given alone

II. Facilitation of metabolism of the more potent anesthetic which may increase the risk of nephrotoxicity

III. Reduction in the anesthetic requirement fot he more potane anesthetic due to a second gas effect.

A. I only
B. II only
C. I and III
D. II and III
E. I. II and III

368. What is responsible for the very short duration and very rapid onset of effect of the so-called Ultra short acting
barbiturates?

A. The presence of a Sulfur group

B. The presence of Methoxy group

C. The presence of substituted barbital structure

D. The high lipid solubility of these agents

E. The increased in the number of polar substituents

369. which of the following intravenous agents can only be used to induced anesthesias but not to maintain anesthesia?

I. Propofol

II. Thiopental

III. Ketamine

A. I only
B. II only
C. I and III
D. II and III
E. I, II and III

370. Which of the following anesthetic is associated with a Dissociative state characterized by catatonia, amnesia, and
analgesia with or without actual loss of consciousness?

A. Propofol

B. Thiopental

C. Midazolam

D. Ketamine

E. Nitrous Oxide

371. Which of the following characteristics is consistent with the drug Propofol?

I. Causes a marked reduction in blood pressure during induction

II. Recovery from anesthesia is rapid

III. Post-operative nausea and vomiting is minimal due to its anti-emetic effect

A. I only
B. II only
C. I and III
D. II and III
E. I, II and III

372. What is the common mechanism of action of the Local anesthetics when given in their usual doses?

Downloaded by ;%'%SiLeNt'%; (jemribsagpatan123@gmail.com)


lOMoARcPSD|5190755

A. Inhibition of Ligand gated sodium channel

B. Inhibition of the voltage gated sodium channel

C. Stimulate outward potassium flux

D. Stimulate calcium extrusion from the cell

E. Inhibit the sodium, potassium ATPase Pump

373. Which of the following Local anesthetics is hydrolyzed to Para-amino benzoic acid?

A. Cocaine

B. Isobucaine

C. Lidocaine

D. Bupivacaine

E. Tetracaine

374. Allergic reaction is most likely to occur with which of the following agents?

A. Isobucaine

B. Prilocaine

C. Procaine

D. Lidocaine

E. Mepivacaine

375. Which of the following local anesthetics is a substrate for metabolic degradation by the enzyme butyrylcholinesterase?

A. Chlorprocaine

B. Etidocaine

C. Ropivacaine

D. Lidocaine

E. Bupivacaine

376. Which of the following agents is considered to be the most neurotoxic when given as a spinal anesthetics causing a
transient reticular irritation?

A. Procaine

B. Lidocaine

C. Bupivacaine

D. Mepivacaine

E.Prilocaine

377. Cardiotoxocity manifesting as a electromechanical dissociation, cardiovascular collapse and death, is a complication
of therapy most commonly seen with which of the following local anesthetics?

A. Mepivacaine

B. Ropivacaine

C. Lidocaine

D. Procaine

E. Bupivacaine

378. When large doses of a local anesthetic must be administered, the risk of developing seizures can be significantly
reduced with the used of which of the following agents as pre-medication?

A. Midazolam

B. Thiopental

C. Succinyl choline

D. Propofol

E. Diphenhydramine

Downloaded by ;%'%SiLeNt'%; (jemribsagpatan123@gmail.com)


lOMoARcPSD|5190755

379.Which of the following Local anesthetics can predisposed to the development of methemoglobinemia when given in
large doses during regional anesthesia?

A. Procaine

B. Lidocaine

C. Cocaine

D. Tetracaine

E. Prilocaine

380. Which of the following metabolites of local anesthetics when formed in the body in significant amounts can cause
oxidation of the iron in hemoglobin in the ferric form?

A. Glycinexylidide

B. Ortho-toluidine

C. para amino benzoic acid

D. Benzoic acid

E. Demethylamine

381. Which of the following anti-psychotics drug groups primarily inhibit the Dopamine-2 receptors in the brain?

I. Phenothiazines

II. Dibenzodiazepines

III. Butyrophenones

A. I only
B. II only
C. I and III
D. II and III
E. I, II and III

382. In terms of potency, which of the drugs listed below is expected to be the most potent?

A. Trifluorophenazine

B. Thioridazine

C. Piperacetazine

D. trifluopromazine

E. Chlorpromazine

383. The Anti-psychotic drug Clozapine belongs to what chemical class of agents?

A. Dibenzoxazepine

B. Dibenzodiazepine

C. Benzisoxazole

D. Fluorobenzyindole

E. Thienobenzodiazepine

384. Which of the following agents is expected to produce the effects and side effects comparable to Haloperidol?

a. Mesoridazine

B. Chlorprothixene

C. Thiothixene

D. Promazine

E. Fluphenazine

385. Anticholinergic effect manifesting as loss of accommodation, difficulty in urination, constipation, and a toxic
confusional state is most commonly observed with which of the following drugs?

A. Haloperidol

B. Sertindole

C. Chlopromazine

Downloaded by ;%'%SiLeNt'%; (jemribsagpatan123@gmail.com)


lOMoARcPSD|5190755

D. Loxapine

E. Chlorpropthixene

386. Dystoinic reactions such as an oculogyric crisis and pseudoparkinsonism such as tremors are side-effects most likely
to be seen in patients taking which of the following medications?

A. Thioridazine

B. Chlorpromazine

C. Loxapine

D. Trifluphenazine

E. Molindone

387. Neuroleptic malignant syndrome which resembles malignant hyperthermia is more commonly seen with which of the
following agents?

A. Mesoridazine

B. Promazine

C. Loxapine

D. Haloperidol

E. Sertindole

388. Which of the following drugs for psychosis is most commonly associated with prolongation of QT interval on ECG
which can lead to fatal arrhythmias?

A. Thioridazine

B. Mesoridazine

C. Haloperidol

D. Clozapine

E. Loxapine

389. Which of the following drugs when given to patients will necessitate weekly WBC monitoring for the first 6 months of
therapy and every 3 weeks thereafter due to its propensity to cause agranulocytosis?

A. thioridazine

B.Chlopromazine

C. Loxapine

D. Clozapine

E. Molindone

390. Which of the following agents is most likely to cause deposits in the retina which can lead to blindness necessitating a
baseline and regular ophthalmologic examination when administered?

A. Thioridazine

B. Chlorpromazine

C. Loxapine

D. Trifluphenzaine

E. Molindone

391. Weight gain is expected complication of therapy with all the clinically useful antipsychotics. One of the drugs listed
below, however does not produce this side effect. What is this drug?

A. Thioridazine

B. Chlorpromazine

C. Loxapine

D. Trifluphenazine

E. Molindone

392. Tardive dyskinesia is considered as the most important unwanted effect of antipsychotic agents. What is considered
as the mechanism of development of this condition?

Downloaded by ;%'%SiLeNt'%; (jemribsagpatan123@gmail.com)


lOMoARcPSD|5190755

I. Dopamine receptor block

II. Relative cholinergic deficiency

III. Dopamine receptor hypersensitivity

A. I only
B. II only
C. I and III
D. II and III
E. I, II and III

393. Which of the following drugs is most commonly associated with seizures?

A. Thioridazine

B. Chlorpromazine

C. Loxapine

D. Trifluphenazine

E. Molindone

394. Poisonings with antipsychotics are rarely fatal. Which of the following agents is associated with a higher incidence of
fatal poisoning?

A. Mesoridazine

B. Chlorpromazine

C. Trifluphenazine

D. Haloperidol

E. Molindone

395. What is the most common side effect associated with the used of tricyclic antidepressants?

A. Sedation

B. Tachycardia

C. Weight gain

D. Orthostatic hypotension

E. Anticholinergic effect

396. Which of the following agents is classified as a tricyclic antidepressant or 1 st generation anti-depressants?

A. Bupropion

B Trazodone

C.Desipramine

D. Amoxapine

E. Mirtazepine

397.Which of the following is/are advantage/s of the SSRI’s compared to the rest of the drugs used for depression?
I. Lesser lethality with overdose

II. Generally no dose titration is required

III. Minimal anticholinergic effects

A. I only
B. II only
C. I and III
D. II and III
E. I, II and III

398. A psychiatric patient on medications develops tremor, thyroid enlargement, and leukocytosis. The drug he is taking is
most likely

A. Chlorpromazine

B. Lithium

C. Desipramine

Downloaded by ;%'%SiLeNt'%; (jemribsagpatan123@gmail.com)


lOMoARcPSD|5190755

D. Fluoxetine

E. Haloperidol

399. Serotonin syndrome which resembles malignant hyperthermia in symptomatology occurs when SSRI’s like Fluoxetine
and Sertraline are given concurrently with

I. TCA’s like Nortriptyline

II. MAOI’s like Phenelzine

III. Heterocyclics like Maprotiline

A. I only
B. II only
C. I and III
D. II and III
E. I, II and III

400. MAO-A is the amine oxidase primarily responsible for the metabolism of Norepinephrine, Serotonin, and tyramine,
while MAO-B is the specific for the metabolism of Dopamine. Which of the following agents is a selective reversible MAO-A
inhibitor whose advantage is a lesser risk of causing hypertensive crisis with tyramine foods?

A. Selegiline

B. Nefazodone

C. Moclobemide

D. Trazodone

E. Imipramine

401. Which of the following conditions enhance the toxicity of lithium?

I. Use of thiazide diuretics

II. Diarrhea and vomiting

III. Excessive sweating

A. I only
B. II only
C. I and III
D. II and III
E. I, II and III

402. Imipramine has been found to be useful in the treatment of nocturnal enuresis ( bed wetting). This clinical use
explained by which of the following effects of the drug?

A. Inhibition of reuptake of norepinephrine

B. Inhibition reuptake of serotonin

C. Anticholinergic effects

D. Sedating effect

E. Inhibition of reuptake of Dopamine

403. Which of the following findings is/are consistent with the poisoning of antipsychotic drugs?

I. Hypotension

II. Hypothermia

III. Ventricular arrythmias

A. I only
B. II only
C. I and III
D. II and III
E. I, II and III

404. Which of the following agents interventions is/are appropriate in the management of poisoning with the Phenothiazine
antipsychotic?

I. Gastric lavage is unnecessary one hour after ingestion of tablets

II. Activated charcoal is given followed by saline cathartics

III. Epinephrine is the preferred pressor agent for persistent hypertension

Downloaded by ;%'%SiLeNt'%; (jemribsagpatan123@gmail.com)


lOMoARcPSD|5190755

A. I only
B. II only
C. I and III
D. II and III
E. I, II and III

405. Which of the following agents are avoided in patients taking Tranylcypromine?

I. Ephedrine

II. Phenylpropanolamine

III. Levodopa

A. I only
B. II only
C. I and III
D. II and III
E. I, II and III

406. Which of the following agents can produce the so-called serotonin syndrome when given to a patient already on
Phenelzine?

I. Venlafaxine

II. Buspirone

III. Fluoxetine

A. I only
B. II only
C. I and III
D. II and III
E. I, II and III

407.Which of the following side-effects of Lithium can be seen even in usual doses thus by itself is NOT considered a sign
of Lithium toxicity ?

A. Coarse hand tremors

B. Polyuria

C. hyperreflexia

D. Confusion

E. ataxia

408. Which of the following may be considered as mechanism/s of effect of Beta lactam antibiotics?

I. Inhibition of activity of the transpeptidase enzyme

II. Inhibition of formation of the 50s ribosomal subunit

III. Binding to the so-called PBPs and related proteins in the cell membrane

A. I only
B. II only
C. I and III
D. II and III
E. I, II and III

409. Which of the following statements is/are correct regarding Penicillins?

I Penicillins are not metabolized extensively by liver enzyme

II. Probenecid increases Penicillin excretion by blocking renal tubule reabsorption

III. Ampicillin undergoes enterohepatic cycling

A. I only
B. II only
C. I and III
D. II and III
E. I, II and III

410. Which of the following statements in Sulfonamides is/are correct?

I. Their effect is attributed to their ability to inhibit the enzyme dihydrofolate reductase

II. As sine agents, they are bacteriostatic but are bactericidal when given with Trimethoprim

Downloaded by ;%'%SiLeNt'%; (jemribsagpatan123@gmail.com)


lOMoARcPSD|5190755

III. They can cause hemolytic anemia if given to G6PD deficient individuals

A. I only
B. II only
C. I and III
D. II and III
E. I, II and III

411. Which of the following may be an expected complication with the use of Aminoglycosides?

I. Neuromuscular blockade

II. Thrombotic Thrombocytopenic purpura

III. Inhibition of the CYP3A4 enzyme

A. I only
B. II only
C. I and III
D. II and III
E. I, II and III

412. Which of the following agents is most effective against nosocomial infection with Pseudomonas aeruginosa?

A. Clarithromycin

B. Ceftazidime

C. Amikacin

D. Imipenem

E. Metronidazole

413. Which of the following drugs can be given orally to treat infections caused by Peudomonas aeruginosa?

A. Ciprofloxacin

B. Ceftazidime

C. Piperacillin

D. Clindamycin

E. Azithromycin

414. Which of the following agents can be safely used for a pregnant women with urinary tract infection?

A. Cotrimixazole

B. Ampicillin

C. Norfloxacin

D. Doxycycline

E. Chloramphenicol

415. Which of the following statements correctly describes the properties of Aminoglycosides?

I. They are mainly bacteriostatic

II. They are mainly effective against gram-negative aerobic bacteria

III. They inhibit protein synthesis

A. I only
B. II only
C. I and III
D. II and III
E. I, II and III

416.Which of the following agents can penetrate inflamed meninges in sufficient concentration to be usedful for meningitis?

A. Amikacin

B. Tobramycin

C. Chloramphenicol

D. Aztreonam

E. Gentamicin

Downloaded by ;%'%SiLeNt'%; (jemribsagpatan123@gmail.com)


lOMoARcPSD|5190755

417. Which of the following agents has acivity against anaerobic organisms against most protozoan infection?

A. Clindamycin

B. Metronidazole

C. Chloramphenicol

E. Doxycycline

418. Which of the following drugs attacks plasmodiain exoerythrocytic stages and is an effective prophylactic agent against
Plasmodium vivax and Plasmodium ovale?

A. Quinine

B. Chloroquine

C. Primaquine

D. Sulfadoxine

E. Pyrimethamine

419.Which of the following agents is bactericidal in its activity?

A. Clindamycin

B. Erythromycin

C. Chloramphenicol

D. Doxycycline

E. Ciprofloxacin

420.Which of the following agents exhibit a primary concentration dependent bactericidal activity wherein the rate and
extent of killing increases with increasing concentrations of drug?

A. Amikacin

B. Vancomycin

C. Clindamycin

D. Ceftriaxone

E. Piperacillin

421.Among patients with renal impairment, the dose of antibiotics must be adjusted according to the creatinine clearance.
Which of the following drugs does not require such adjustment in dosing for patients with decreased creatinine clearance?

A. Imipenem

B. Penicillin G

C. Nafcillin

D. Ceftazidime

E. Cefuroxime

422. Which of the following agents requires adjustment or may even have to be avoided in patients with hepatic
impairment?

I. Erythromycin

II. Metronidazole

III. Clindamycin

A. I only
B. II only
C. I and III
D. II and III
E. I, II and III

423. Which of the following agents is contraindicated in patients less than 18 years of age and pregnant patients due to its
adverse effects on cartilage development?

A. Sulfamethoxazole

B. Ethambutol

Downloaded by ;%'%SiLeNt'%; (jemribsagpatan123@gmail.com)


lOMoARcPSD|5190755

C. Chloramphenicol

D. Doxycycline

E. Ciprofloxacin

424. Which of the following antibiotics is contraindicated in children less than 9 years of age and in pregnant patients due
to the risk of permanent staining of teeth and enamel dysplasia in children?

A. Sulfamethoxazole

B. Ethambutol

C. Chloramphenicol

D. Doxycycline

E. Ciprofloxacin

425.Which of the following agents may predispose the neonate premature babies to kernicterus when given to this subset
of patients?

A. Sulfamethoxazole

B. Ethambutol

C. Chloramphenicol

D. Doxycycline

E. Ciprofloxacin

426. What is the primary role of agents such as Tazobactam amd Sulbactam in antibacterial therapy?

A. Increase in the oral bioavavilability of the Penicillins

B. Effective against most gram-negative aerobic bacteria

C. Effective against anaerobic bacteria including Clostridia

D. Minimize destruction of the Penicillins by bacterial beta-lactamases

E. Improve CNS penetrability of Penicillins

427.Which of the following antibiotic combinations is associated with clinically significant antimicrobial antagonism which
has been reported to cause higher rate of mortality in patients given this combination compared to patients receiving any of
the agent singly?

A. Penicillins + Cephalosporin

B. Penicillin + aminoglycoside

C. Erythromycin + aminoglycoside

D. Penicillin + Chloramphenicol

E. Erythromycin + Doxycycline

428.Which of the following is the preferred empiric treatment for patients suspected or proved to have sysytemic fungal
infection of any etiology?

A. Itraconazole

B. Amphotericin B

C. Fluconazole

D. Flucytosine

E. Ketoconazole

429. The activity of Penicillin G is defined units. How many units does each milligram of crystalline sodium penicillin G
contain?

A. 1

B. 58

C. 180

D. 1600

E. 2600

Downloaded by ;%'%SiLeNt'%; (jemribsagpatan123@gmail.com)


lOMoARcPSD|5190755

430. The absorption of most oral penicillins is impaired by food. An exception to this is:

A. Ampicillin

B. Amoxicillin

C. Penicillin

D. Penicillin G

E. Oxacillin

431. Which of the following organism is/are susceptible to Penicillin G, making the drug the drug of choice in the treatment
of the infection caused by this/these organism?

I. Treponema pallidum

II.Pneumococcus

III. Beta-hemolytic streptococci

A. I only
B. II only
C. I and III
D. II and III
E. I, II and III

432. The so-called Isoxazolyl penicillins are primarily indicated for the treatment of infection caused by what organism?

A. Staphylococcus aureus

B. Bacteroides fragilis

C. Escherichia coli

D. Haemophilus influenzae

E. Streptococcus peumoniae

433. Penicillins classified as ureido penicillins have extended spectrum of activity against gram-negative bacteria
compared to the natural and other semisynthetic penicillins. Which of the following organism are the ureido penicillins
effective against?

I. Enterococci

II. Klebsiella pneumoniae

III. Pseudomonas aeruginosa

A. I only
B. II only
C. I and III
D. II and III
E. I, II and III

434. Which of the following antibiotics belong to the ureido penicillin class of agents?

A. Carbenicillin

B. Ticarcillin

C. Peparicillin

D. Bacampicillin

E. Pivampicillin

435. Which of the following agents is a first generation cephalosporin that is given intravenously?

A. Cefazolin

B. Cephalexin

C. Cephadrine

D. Cefadroxil

E. Cephapirin

436. The second generation cephalosporin includes the Cephamycins. What is the distinctive advantage of the
cephamycin over the rest of the second generation cephalosporin?

A. they have greater selectivity against Staphylococcus aureus

Downloaded by ;%'%SiLeNt'%; (jemribsagpatan123@gmail.com)


lOMoARcPSD|5190755

B. They have average for anaerobic bacteria

C. They are effective against MRSAs

D. They have greater spectrum of activity including Peudomonas aeruginosa

E. They can be given both orally and parenterally

437. Which of the following second generation cephalosporins can be given orally and parenterally?

A. Cefaclor

B. Cefuroxime axetil

C. Cefotetan

D. Cefprozil

E. Cefoxitin

438. Which of the following agents belong the cephalosporins group called cephamycins?

A. Cefamandole

B. Cefprozil

C. Cefoxitin

D. Loracarbef

E. Cefaclor

439. Which of the following third generation cephalosporins have activity against Pseudomonas aeruginosa?

I. Ceftriaxone

II. Ceftazidime

III. Cefoperazone

A. I only
B. II only
C. I and III
D. II and III
E. I, II and III

440. Which of the following agents is considered as fourth generation cephalosporin?

A. Cefepime

B. Cefotaxime

C. Cefoperazone

D. Cefixime

E. Moxolactam

441. Which of the following cephalosporin is/are associated with Disulfiram-effect when alcohol is taken by patients who
are on this/these antibiotics?

I. Cefotetan

II. Ceftazidime

III. Cefamandole

A. I only
B. II only
C. I and III
D. II and III
E. I, II and III

442. Which of the following cephalosporins is associated with hypoprothrombinema and has increased risk of bleeding
disorders?

A. Cefoperazone

B. Cefoxitin

C. Cefuroxime

D. Ceftriaxone

Downloaded by ;%'%SiLeNt'%; (jemribsagpatan123@gmail.com)


lOMoARcPSD|5190755

E. Cefotaxime

443. Which of the following agents is considered as first line drug in the management of neisseria gonorrhea infection?

A. Ceftriaxone

B .Cefuroxime

C.Penicillin G

D. Amoxicillin

E. Trimethoprim-Sulfamethoxazole

444. Which of the following Beta-Lactam antibiotic is called a monobactam?

A. Moxolactam

B. Aztreonam

C. Meropenem

D. Loracarbef

E. Ticarcillin

445. What is the use of Cilastatin?

A. Effective agent against Pseudomonas aeruginosa

B. It prevents tubular secretion of Imipenem

C. It prevents glomerula filtration of Imipenem

D. It inhibits the renal dehydropeptidase which metabolizes Imipenem

E. It inhibits liver dehydropeptidase which metabolizes Imipenem

446. Which of the following antibacterial agents primarily work by inhibiting cell wall synthesis?

I.Polymyxin B

II. Vancomycin

III. Streptpgramins

A. I only
B. II only
C. I and III
D. II and III
E. I, II and III

447. Which of the following agents primarily work by inhibiting aminoacyl translocation and blocking the formation of
initiation complexes?

A. Aminoglycosides

B. Chloramphenicol

C. Linezolid

D. Tetracyclines

E. Macrolides

448. Which of the following drugs reversibly binds to the 30S subunit of the bacterial ribosome and inhibits the peptidyl
transferase step of the protein synthesis?

A. Aminoglycosides

B. Chloramphenicol

C. Linezolid

D. Tetracyclines

E. Macrolides

449. Which of the following agents reversibly binds to the 30S subunits of the bacterial ribosome blocking the binding of
aminoacyl-tRNA to the acceptor site on the mRNA-ribosome complex?

A. Aminoglycosides

B. Chloramphenicol

Downloaded by ;%'%SiLeNt'%; (jemribsagpatan123@gmail.com)


lOMoARcPSD|5190755

C. Linezolid

D. Tetracyclines

E. Macrolides

450. (MISSING)

451. Which of the following agents inhibits protein synthesis by binding at the 23S ribosomal RNA and preventing the
formation of ribosome complex that initiates protein snthesis?

A. Aminoglycosides

B. Chloramphenicol

C. Linezolid

D. Tetracyclines

E. Macrolides

452. The mechanism of action of Clindamycin is similar to the mechanism of action of which of the following agents?

A. Aminoglycosides

B. Chloramphenicol

C. Linezolid

D. Tetracyclines

E. Macrolides

453. Which of the following statements correctly describes the properties of the drugs Minocycline and Doxycycline?

I. They have the longest half-lives among the Tetracyclines

II. Their absorption is not impaired by divalent cations like calcium and magnesium

III. Their absorption is not impaired by food

A. I only
B. II only
C. I and III
D. II and III
E. I, II and III

454. Which of the following tetracyclines is the drug of choice when tetracyclines are needed in a patient with renal
insufficiency??

A. Oxytetracycline

B. Chlortetracycline

C. Methacycline

D. Doxycyclie

E. Minocycline

455. Which of the following macrolides can inhibit cytochrome P450 enzymes can cause an increase in the serum
concentrations of drugs like theophylline and warfarin?

I. Erythromycin

II. Azithromycin

III. Clarithromycin

A. I only
B. II only
C. I and III
D. II and III
E. I, II and III

456. What form of erythromycin has the best oral absorption?

A. Erythromycin base

B. Erythromycin ethylsuccinate

C. Erythromycin estolate

Downloaded by ;%'%SiLeNt'%; (jemribsagpatan123@gmail.com)


lOMoARcPSD|5190755

D. Erythromycin lactobionate

E. Erythromycin stearate

457. What is the clinical use of the tetracycline drug Demeclocycline?

A. Prophylaxis against plague

B. Antiprotozoal agent

C. Used to eradicate meningococcal carrier state

D. Management of SIADH

E. Management of Nephrogenic Diabetes Insipidus

458. Which of the following is/are clinical indication/s for the drug Clindmycin?

I. Treatment of Severs gram-negative bacterial infection

II. Treatment of severe anaerobic infections caused by bacteroides

III. Prophylaxis of endocarditis in patients with valvular heart disease undergoing dental procedures

A. I only
B. II only
C. I and III
D. II and III
E. I, II and III

459. Which of the following may be considered as indication for the streptogramin combination Quinupristin-Dalfopristin?

I. Treatment of MRSA

II. Treatment of Multi-Drug resistant strains of Streptococci

III. Treatment of Penicillin-resistant strain of Streptococcus pneumonia

A. I only
B. II only
C. I and III
D. II and III
E. I, II and III

460. Which of the following statements correctly describes the characteristics of Aminoglycosides?

I. Exhibit synergistic activity with beta-lactam antibiotics

II. May be given both orally and parenterally for systemic activity

III. Readily penetrate the blood brain barrier reaching adequate concentrations in the CSF

A. I only
B. II only
C. I and III
D. II and III
E. I, II and III

461. Which of the following drugs may enhance toxicity of Aminoglycosides if given with the latter concurrently to a patient?

I.Penicillin G

II. Furosemide

III. Vancomycin

A. I only
B. II only
C. I and III
D. II and III
E. I, II and III

462. Which of the following Aminoglycosides as the most ototoxic in terms of tis side-effect on the cochlear apparatus?

A. Neomycin

B. Amikacin

C. Streptomycin

D. Tobramycin

E. Netilmicin

Downloaded by ;%'%SiLeNt'%; (jemribsagpatan123@gmail.com)


lOMoARcPSD|5190755

463. Which of the following Aminoglycosides is considered as the most vestibulotoxic?

A. Neomycin

B. Amikacin

C. Streptomycin

D. Tobramycin

E. Netilmicin

464. Which of the following Aminoglycosides is considered as the most nephrotoxic?

A. Neomycin

B. Amikacin

C. Streptomycin

D. Tobramycin

E. Netilmicin

465. Which of the following disease-causing organism is/are the aminoglycosides most useful either assingle agents or in
combinations?

I. Mycobacterium tuberculosis

II.Mycobacterium avium intracellulare

III. Staphylococcus aureus

A. I only
B. II only
C. I and III
D. II and III
E. I, II and III

466. Which of the following agents is given with the Sulfonamides for its synergistic effect with the latter or inhibiting folic
acid synthesis in protozoal cells than mammalian or bacterial cells?

A. Pyrimethamine

B. Trimethoprim

C. Silver

D. Mafenide

E. Salicylic acid

467. What is the mechanism of action of quinolone antibacterial agents?

A. Inhibit the incorporation of GABA into Dihydropteroate

B. Inhibit the bacterial enzyme Topoisomerase II

C. Inhibit the DNA dependent RNA polymerase of bacteria

D. Inhibit the Transpeptidation process in bacterial cellwall synthesis

E. Inhibit bacterial dihydrofolate reductase

468. A patient receiving sulfadiazine 4 grams per day may benefit with co-administration of which of the following agents as
preventive therapy of sulfadiazine-related complications?

I. Sodium bicarbonate

II. Ammonium chloride

III Folinic acid

A. I only
B. II only
C. I and III
D. II and III
E. I, II and III

469. Which of the following drugs given for tuberculosis is/are bactericidal in activity at the usual dose given for TB?

Downloaded by ;%'%SiLeNt'%; (jemribsagpatan123@gmail.com)


lOMoARcPSD|5190755

I. Ethambutol

II. Rifampicin

III. Isoniazid

A. I only
B. II only
C. I and III
D. II and III
E. I, II and III

470. A 3 year old boy is diagnosed with primary tuberculosis infection. The appropriate anti-TB regimen for the child may
include which of the following agents?

I. Ethambutol

II. Rifampicin

III. Isoniazid

A. I only
B. II only
C. I and III
D. II and III
E. I, II and III

471. Which of the following drugs given for tuberculosis is metabolized by the liver enzyme N-acetyltransferase whose
expression is genetically determined may vary in different populations?

A. Isoniazid

B. Rifampicin

C. Ethambutol

D. Pyrazinamide

E. Streptomycin

472. Which of the following TB drugs is given only during the intensive phase of therapy?

A. Isoniazid

B. Rifampicin

C. Ethambutol

D. Pyrazinamide

E. Streptomycin

473. What is the acceptable dose for isoniazid in the treatment of TB?

A. 1 mg/kg/day

B.5mg/k/day

C. 10mg/k/day

D. 15mg/k/day

E.20mg/k/day

474. What is the dose for rifampicin in the treatment of TB?

A. 1 mg/kg/day

B.5mg/k/day

C. 10mg/k/day

D. 15mg/k/day

E.20mg/k/day

475. Which of the following agents may be considred as useful 2 nd line agents for TB?

I. Kanamycin

II. Amikacin

III. Ciprofloxacin

Downloaded by ;%'%SiLeNt'%; (jemribsagpatan123@gmail.com)


lOMoARcPSD|5190755

A. I only
B. II only
C. I and III
D. II and III
E. I, II and III

476. Which of the following agents given for tuberculosis may provoke an attack of acute gouty arthritis?

A. Isoniazid

B. Rifampicin

C. Ethambutol

D. Pyrazinamide

E. Streptomycin

477. Which of the following drugs has been historically associated with the development of Creutzfeldt-Jakob disease, a
fatal neurodegenerative disease caused by prions?

A. Pancreas derived insulin

B. Pituitary derived growth hormone

C. Thyroid hormone extract

D. Recombinant growth hormone

E. Cortisol

478. Ocreotide is a drug used for the treatment of such conditions as acromegaly, gastrinoma, and glucagonoma. What
hormone is ocreotide an analog of?

A. Somatostatin

B. Growth hormone

C. Prolactin

D. Oxytocin

E. Dopamine

479. Which of the following correctly describes the nomal physiologic hormone release?

I. GnRH is released continuously by the hypothalamus to induce secretion of FSH and LH by the pituitary

II. ACTH release occurs in pulses that peak iin the early morning hours and after meals

III. The release of cortisol is constant throughout the day maintaining a level that varies little with meals

A. I only
B. II only
C. I and III
D. II and III
E. I, II and III

480. Which of the following is/are appropriate indications for oxytocin?

I. Control of postpartum uterine bleeding

II. Stimulate initial milk production

III. Induction of labor

A. I only
B. II only
C. I and III
D. II and III
E. I, II and III

481. What is the primary use of the drug Desmopressin?

I. Management of nephrogenic Diabetes insipidus

II. Management of Central Diabetes Insipidus

III. Control of gastrointestinal bleeding

A. I only
B. II only
C. I and III

Downloaded by ;%'%SiLeNt'%; (jemribsagpatan123@gmail.com)


lOMoARcPSD|5190755

D. II and III
E. I, II and III

482. Which of the following statements correctly describes the thyroid hormones?

I. T3 is better absorbed after oral administration than T4

II. Peripheral deiodination of T3 converts it to more active T4

III. Hypothyroidism increases the clearance of both hormones leading to an decrease in their half-lives

A. I only
B. II only
C. I and III
D. II and III
E. I, II and III

483. Which of the following agents may interfere with the absorption of levothyroxine that patient must be advised on the
proper interval of intake with this/these agents?

I. Sucralfate

II. Ferrous sulfate

III. Aluminum hydroxide

A. I only
B. II only
C. I and III
D. II and III
E. I, II and III

484. A 43 year old female who works in a hospital was admitted for tachycardia, palpitations and weight loss. Her T4 was
elevated with suppresses TSH but her thyroid scan showed a normal-sized gland with normal iodine uptake. Which of the
following should be suspected?

A. Presence of Grave’s disease

B. Presence of solitary toxic nodule

C. Iodine deficiency goiter

D. Illicit intake of Levothyroxine

E. Hypothyroidism

485. A patient is receiving 50mg of Propylthiouracil three times a day. If the plan is to shift her medication to methimazole,
what dose of methimazole should you recommended?

A. 5 mg once daily

B. 15 mg once daily

C. 30 mg once daily

D. 45 mg once daily

E. 60 mg once daily

486. In monitoring response to therapy with methimazole, which of the following findings taken 3 months after initiating
therapy best indicates good response to therapy?

A. a normal TSH

B. a normal T4

C. a lower than normal T4

D. a lower than normal TSH

E. a higher than normal TSH

487. Which of the following describes the mechanism of action of the thioamides for the management of hyperthyroidism?

I. Block uptake of iodine by the follicular cells

II. Block the peroxidase-catalyzed reactios

III. Block coupling of iodotyrosines

A. I only
B. II only
C. I and III

Downloaded by ;%'%SiLeNt'%; (jemribsagpatan123@gmail.com)


lOMoARcPSD|5190755

D. II and III
E. I, II and III

488. Which of the following interventions may be appropriate to monitor or treat thionamide-associated agranulocytosis?

I. Discontinuing the offending drug usually reverses the side-effect

II. Monitoring the WBC is necessary for the first three months of therapy

III. Switching of therapy from one thionamide to another may reverse the effect

A. I only
B. II only
C. I and III
D. II and III
E. I, II and III

489. Which of the following cautions must be observed when iodides are given to treat hypothyroidism?

I. Iodides must be initiated before thionamide therapy

II.Iodides must be avoided if treatment with radioactive iodine is likely

III. Iodides are the preffered drugs for chronic therapy during pregnancy

A. I only
B. II only
C. I and III
D. II and III
E. I, II and III

490. What is the role of beta-blockers in the management of thyrotoxicosis?

I. beta-blockers can inhibit the peroxidase-catalyzed reaction in the synthesis of thyroid hormones

II. Beta-blockers control the sympathetic symptoms of hyperthyroidism

III. Beta-blockers may inhibit the peripheral conversion of T4 to T3

A. I only
B. II only
C. I and III
D. II and III
E. I, II and III

491. Which of the following statements correctly describes the thyroid hormone replacement therapy?

I. Infants and children with congenital hypothyroidism require higher dose per kilogram body weight than adults

II. Steady state levels of thyroxine takes about 6-8 weeks to achieve after initiating therapy

III. Older adults and those with long-standing disease must be started on lower than usual dose of levothyroxine

A. I only
B. II only
C. I and III
D. II and III
E. I, II and III

492. A 20-year old female is receiving Hydrocortisone 20 mg IV three times a day. If the plan is to discharge the patient
with an equivalent dose of Oral Prednisone, what dose of Prednisone will you recommend?

A. 5 mg once a day

B. 10mg once a day

C. 10 mg in the morning and 5 g in the afternoon

D. 15 mg in the morning and 10 mg in the afternoon

E. 20 mg in the morning and 15 mg in the afternoon

493. What is the physiologic dose of glucocorticoid?

A. Hydrocortisone dose less than 10-20 mg/day

B. Hydrocortisone dose greater than 10-20mg/day

C. Prednisone dose ranging from 10-20mg/day

D. Prednisone dose less than 10-20mg/day

E. Prednisone dose less than 10-20mg/day

Downloaded by ;%'%SiLeNt'%; (jemribsagpatan123@gmail.com)


lOMoARcPSD|5190755

494. Which of the following effects is/are consistent when cortisol in the body is less than 10 mg/day?

I. Metabolism of carbohydrate, fat and proteins

II. Inhibition of release of cytokines from macrophages

III. Enhanced vascular and bronchial smooth muscle response to cathecolamiines

A. I only
B. II only
C. I and III
D. II and III
E. I, II and III

495. Which of the following drugs or drug combinations is appropriate maintenance therapy for patient with primary adrenal
insufficiency?

A. Hydrocortisone + Fludrocortisone

B. Dexamethasone + Hydrocortisone

C. Hydrocortisone

D. Prednisone

E. Betamethasone

496. Which of the following is/ are appropriate indications for using Glucocorticosteroids?

I. In mothers who may have premature labor and delivery before 34 weeks of gestation

II. In patients with secondary pulmonary tuberculosis prior to initiation of antitubercular therapy

III. In patients who have recently undergone organ transplantation

A. I only
B. II only
C. I and III
D. II and III
E. I, II and III

497. In patients on glucocorticoid therapy, which of the following monitoring plans sould be initiated?

I. Blood pressure monitoring

II. Bone density monitoring

III. Blood sugar monitoring

A. I only
B. II only
C. I and III
D. II and III
E. I, II and III

498. Which of the following statements is/are correct regarding adrenal suppression as a complication of therapy with
glucocorticosteroids?
I. It is expected to occur when glucocorticosteroids therapy is extended beyond 2 weeks

II. Dosage reduction when the therapy is to stopped must be done very slowly when the dose reaches replacement levels

III. Patients must receive supplementary therapy at times of stress like surgery or trauma

IV. Recovery of the hypothalamic-pituitary-adrenal axis is rapid after discontinuing chronic use of Dexamethasone

V. Treatment with ACTH reduces the time required for the return of normal adrenal function

A. I and III

B. II and IV

C. I, II and III

D. I, III and V

E. I, II ,III, IV and V

499. Which of the following glucocorticosteroids is employed in the work-up of patients with Cushing’s Syndrome to
diagnose the most probable etiology of the condition?

A. Hydrocortisone

B. Prednisone

Downloaded by ;%'%SiLeNt'%; (jemribsagpatan123@gmail.com)


lOMoARcPSD|5190755

C. Cortisone

D. Fludprednisolone

E. Dexamethasone

500. What is the Primary role if Zinc in commercial insulin preparations?

A. Improve stability and shelf-life of insulin

B. Improve the solubility of insulin

C. Enhance the activity of insulin in receptors

D. Facilitate absorption of insulin from subcutaneous tissue

E. Enhance the activity of suspending agents

501. The transport of glucose that is facilitated by insulin is mediated by which of the following transporters?
A. GLUT1

B. GLUT2

C. GLUT3

D. GLUT4

E. GLUT 5

502. What is the primary advantage of Insulin Lispro compared to regular Insulin?

A. Longer duration of action reaching up to 6 hours

B. No associated peak levels

C. It is less immunogenic

D. Less likelihood to cause hypoglycemic episodes

E. Rapid onset of action in 5 to 15 minutes

503. Which of the following new insulin preparations has a characteristics release pattern that shows no peak and a
plateau serum insulin level that is maintained for about 24 hours?

A. Insulin Glargine

B. Insulin Lispro

C. insulin Aspart

D. Ultralente Insulin

E. NPH Insulin

504. Which of the following statements is/are correct regarding Lente Insulin?

I. It is a mixture of 30% semilente and 70% ultralente Insulin

II. It is also known as NPH

III. It is an intermediate-acting Insulin preparation

A. I only
B. II only
C. I and III
D. II and III
E. I, II and III

505. A 50 year old male patient with Type2 Diabetes mellitus is maintained for the last 7 days on subcutaneous Isophane
Insulin at 20 units and 10 units pre-supper. For the last three days, his capillary blood glucose levels have been
consistently showing the following values: 190 mg% (prebreakfast), 118mg% (2hr-post lunch), and 110 mg% (presupper).
What recommendation will you make regarding his therapy?

A. Increase his pre-breakfast insulin to 24 units while maintaining his pre-supper Insulin at 10 units

B. Increase his pre-breakfast Insulin to 24 units and his pre-supper Insulin to 12 units

C. Maintain his pre-breakfast and pre-supper Insulin doses give Insulin Lispro 4 units pre-breakfast

D. Maintain his pre-breakfast insulin at 20 units while increase his pre-supper insulin to 12 units

E. No change in the insulin doses as all values are within the acceptable range

Downloaded by ;%'%SiLeNt'%; (jemribsagpatan123@gmail.com)


lOMoARcPSD|5190755

506. A 50 year old male patient with type2 diabetes mellitus is maintain for the last 7 days of subcutaneous Isophane
Insulin at 20 Units pre-breakfast and 10 units pre-supper. For the last three days, his capillary blood glucose levels have
been consistently showing the following values: 110mg% ( pre-breakfast), 218mg% (2 hour post breakfast), 110 mg% (2
hour post lunch), and 110 mg% ( pre-supper). What recommendation will you make regarding his therapy?

A. Increase his pre-breakfast insulin to 24 units while maintaining his pre-supper insulin at 10 units

B. Increase his pre-breakfast insulin to 24 units and his pre-supper insulin to 12 units

C. Maintain his pre-breakfast and pre-supper insulin doses but give insulin lispro 4 units pre-breakfast

D. Maintain his pre-breakfast insulin at 20 units while increase his pre-supper insulin to 12 units

E. No change in the insulin doses as all values are withih the acceptable range

507. Immune insulin resistance is said to be due to production of low levels of what type of insulin antibodies?

A. IgA

B. IgD

C. IgE

D. IgG

E. IgM

508. Which of the following oral anti-diabetic drugs is/are known to induce hypoglycemia among diabetic and euglycemic
individuals?

I. Sulfonylureas

II. Biguanides

III. Meglitinides

A. I only
B. II only
C. I and III
D. II and III
E. I, II and III

509. Which of the following antidiabetic drugs produced control of blood sugar by promoting directly or indirectly the
release of insulin by the remaining functional beta cells of the pancreas?

I. Meglitinides

II. Biguanides

III. Thiazolidinediones derivatives

A. I only
B. II only
C. I and III
D. II and III
E. I, II and III

510. Which of the following antidiabetic drugs has the longest half-life?

A. Chlorpropramide

B. Tolazamide

C. Tolbutamide

D. Glyburide

E. Glimepiride

511. What is the recommended clinical use of Repaglinide in the management of Diabetes mellitus?

A. Primarily given to control fasting blood glucose levels

B. Improves tissue response to insulin

C. Control post-prandial hyperglycemia

D. Control overnight blood sugar

E. Prevent absorption of carbohydrates

512. Which of the following drugs is a Biguanide?

Downloaded by ;%'%SiLeNt'%; (jemribsagpatan123@gmail.com)


lOMoARcPSD|5190755

A. Acetohexamide

B. Rosiglitazone

C. Nateglinide

D. Metformin

E. Acarbose

513. In which individuals should Biguanides be avoided as part of therapy for DM because of increased risk of lactic
acidosis?

I. Chronic alcoholics

II. Coronary Artery disease

III. Chronic hepatits

A. I only
B. II only
C. I and III
D. II and III
E. I, II and III

514. Which of the following drugs is involved in regulating the genes involved in lipid and glucose metabolism and adiposite
differentiation by acting as ligand of the PPAR-gamma, thus useful for the managing Insulin resistance?

A. Metformin

B. Repaglinitide

C. Acarbose

D. Tolazamide

E. Rosiglitazone

515. Which of the following drugs may be given to type 1 DM patients as a combination therapy with Insulin?

A. Voglibose

B. Repaglinide

C. Nateglinide

D. Chlorpropamide

E. Glyburide

516. Which of the following are clinical use of Glucagon?

I. Reverse the physical deformity associated with acromegaly

II. Management of severe hypoglycemia

III. Reverse the cardiac effects of beta blockers overdose

A. I only
B. II only
C. I and III
D. II and III
E. I, II and III

517. Which of the following is the common complication associated with the alpha-glucosidase inhibitors?

A. Cardiac arrhythmia

B. Flatulence

C. Congestive heart failure

D. Hepatotoxicity

E. Interstitial Nephritis

518. Which of the following is the most common problem encountered in female patients using Progestin only oral
contraceptive pills?

A. Mastalgia

B. Breakthrough bleeding

Downloaded by ;%'%SiLeNt'%; (jemribsagpatan123@gmail.com)


lOMoARcPSD|5190755

C.Increase in pigmentation

D. Hirsutism

E. hypertension

519. The risk of stroke is especially increase when oral contraceptives are given to which subpopulation of patients?

I. Age above 35 years old

II. Use of progestin only pills

III. Chronic smokers

A. I only
B. II only
C. I and III
D. II and III
E. I, II and III

520. Which of the following drugs is recommended prophylaxis for travelers in malaria endemic areas with known
chloroquine-resistant strains of Plasmodium falciparum?

I. Primaquine

II. Mefloquine

III. Atovaquone-proguanil

A. I only
B. II only
C. I and III
D. II and III
E. I, II and III

521. Although the clinical value of resistance reversal is not established, there are certain drugs that have been shown to
reverse chloroquine-resistance in malaria. Which of the following is an exemple of resistance reversing drugs?

A. Verapamil

B. Propranolol

C. Calcium gluconate

D. Primaquine

E. Diphenhydramine

522. Which of the following is the preferred treatment of malaria caused by Plasmodium vivax or Plasmodium ovale?

A. Clindamycin

B. Doxycycline

C. Fansidar

D. Halofantrine

E. Chloroquine

523. What is the treatment of choice for the management of severe falciparum malaria?

A. Chloroquine phosphate

B. Quinidine gluconate

C. Artemether

D. Artesunate

E. Doxycycline

524. Which of the following agents may be useful for eradicating a carrier state of Entamoeba histolytica?

I. Metronidazole

II. Diloxanide furoate

III. Iodoquinol

A. I only
B. II only
C. I and III

Downloaded by ;%'%SiLeNt'%; (jemribsagpatan123@gmail.com)


lOMoARcPSD|5190755

D. II and III
E. I, II and III

525. Which of the following agents is considered as the standard drug for the initial management of Pneumocystis jiroveci
(carinii) pneumonia in patients with AIDS?

A. Cotrimoxazole

B. Pentamidine

C. Doxycycline

D. Meronidazole

E. Emetine

526. filariasis an infection which can lead to Lymphatic obstructive disease is caused by Wuchreria bancrofti and Brugia
malayi. Eradication of microfilaria can be accomplished by the use of which of the following agents?

A. Praziquantel

B. Diethycarabamazine citrate

C. Niclosamide

D. Albendazole

E. Metronidazole

527. What is the probable mechanism of action of Mebendazole, A broad spectrum anthelmintic agent for Ascariasis,
Trichuriasis, and hookworm infection?

A. Inhibition of the incorporation of PABA in to dehydropteroate

B. Intensifies GABA-mediated transmission in peripheral nerves of nematodes

C. Inhibition of cholinesterase action in parasites leading to paralysis of the worm

D. Inhibition of the microtubule synthesis

E. Increases permeability of the nematode cell membrane to calcium resulting to paralysis and death

528. Which of the following agents is considered as the drug of choice for most cestodal and trematodal infection?

A. Praziquantel

B. Mebendazole

C. Niclosamide

D. Albendazole

E. Metronidazole

529. Which of the following antineoplastic agents is considered as cell cycle specific agent in terms of its action on cancer
cells?

A. Dactinomycin

B. Daunorubicin

C. Vincristine

D. Cisplatin

E. Mechlorethamine

530. Exemestane is a steroidal hormone agent that binds to and irreversibly inactivate aromatase enzyme. What is the
clinical use of the agent?

A. Acute myelogenous leukemia

B. Renal cell carcinoma

C. Choriocarcinoma

D. Breast cancer

E. Colorectal cancer

531. Which of the following correctly provides the sequence of symptoms associated with carbon monoxide poisoning?

A. Psychomotor impairment headache confusion tachycardia coma

Downloaded by ;%'%SiLeNt'%; (jemribsagpatan123@gmail.com)


lOMoARcPSD|5190755

B. Convulsions headache tachycardia loss of visual acuity coma

C. Tachycardia confusion psychomotor impairment loss of visual acuity coma

D. Coma confusion headache tachycardia loss of visual acuity

E. Convulsions loss of visual acuity confusion tachycardia coma

532. Which of the following statements regarding carbon monoxide poisoning is/are correct?

I. The brain and the heart are the most affected organ

II. Forms carboxyhemoglobin which cannot transport oxygen

III. Smokingmales have a higher carboxyhemoglobin level

A. I only
B. II only
C. I and III
D. II and III
E. I, II and III

533. The ability of a chemical agent to cause injury in a given situation or setting is called?

A. Hazard

B. Risk

C. Exposure

D. Toxicity

E. Threshold

534. What is considered as the most common cause of air pollution?

A. Sulfur oxides

B. Nitrogen oxides

C. Hydrocarbons

D. Carbon monoxide

E. Ozone

535. Which of the following aliphatic hydrocarbons is considered as the most hepatotoxic?

A. Carbon monoxide

B. Chloroform

C. Trichloroethylene

D. Tetrachloroethylene

E. Methyl chloroform

536. What is the most important toxidrome of acute exposure to benzene?

A. CNS Stimulation

B. CNS depression

C. Bone marrow injury

D. Peripheral neuropathy

E. Leukemia

537. Which of the following is considered as the most significant serious toxic effect of chronic exposure to benzene?

A. Hepatotoxicity

B. Nephrotoxicity

C. Peripheral neuropathy

D. Bone marrow injury

E. Cardiotoxicity

538. What is considered as the most common cause of industrial lead poisoning?

Downloaded by ;%'%SiLeNt'%; (jemribsagpatan123@gmail.com)


lOMoARcPSD|5190755

A. Absorption of Lead via the gastric mucosa

B. Absorption of lead via the respiratory tract

C. Absorption of lead through the skin

D. Absorption of lead from the antiknock gasoline

E. Absorption of lead from the intestines

539. Which of the following factors predisposed to increased intestinal lead absorption?

I. Increased dietary Calcium

II. Iron deficiency

III. Ingestion in an empty stomach

A. I only
B. II only
C. I and III
D. II and III
E. I, II and III

540. Which of the following statements is/are correct regarding the pharmacokinetics of lead?

I. Lead can cross the placenta and pose a potential hazard to fetus

II. Adults have greater degree of absorption of ingested lead than young children

III. The major route of excretion of lead is through the urine

A. I only
B. II only
C. I and III
D. II and III
E. I, II and III

541. In developing a fetus as well as in a young child, what is considered as the most sensitive target organ for Lead’s
toxic effect?

A. Active bone marrow

B. Kidneys

C. Immature reproductive organs

D. Gastrointestinal tract

E. Developing central nervous system

542. What is the primary repository site of lead in the body of an adult containing about 90% of the total body lead burden?

A. Liver

B. Bone

C. Brain

D. Gastrointestinal tract

E. Skin and its appendages

543. Which of the following chelating agents may be useful in the different phases of management of inorganic lead
poisoning?

I. Calcium disodium edetate

II. Succimer

III. Dimercaprol

A. I only
B. II only
C. I and III
D. II and III
E. I, II and III

544. A patient presenting with encephalopathy, abdominal colic, hemolytic anemia and an elevated liver enzymes several
days after exposure to large amounts of a metallic poison may be suffering from what poisoning?

I. Acute inorganic lead poisoning

Downloaded by ;%'%SiLeNt'%; (jemribsagpatan123@gmail.com)


lOMoARcPSD|5190755

II. Acute Inorganic arsenic poisoning

III. Acute mercury

A. I only
B. II only
C. I and III
D. II and III
E. I, II and III

545. Which of the following effects correctly describe the mechanism of action of arsenic as a poison?

A. Inhibits generation of ATP by binding to cytochrome oxidase system in the electron transport chain

B. Inhibits effective oxygen delivery by oxidizing the hemoglobin iron form ferrous to ferric

C. Inhibits the enzyme activity by binding to the Sulfhydryl groups in the enzyme

D. Alters DNA transcription by impairing synthesis of mRNA

E. Intercalates with the DNA leading to failure of gene expression

546. Hepatolenticular degeneration is a result of chronic exposure to which metal?

A. Lead

B. Mercury

C. Iron

D. Copper

E. Arsenic

547. Chronic excessive exposure to this metal can lead to deposition in various organs and tissues causing the
development of conditions such secondary Diabetes mellitus, restrictive cardiomyopathy, and hepatic failure

A. Lead

B. Mercury

C. Iron

D. Copper

E. Arsenic

548. Erethism, a behavioral pattern characterized by change in mood from shyness, withdrawal and depression with
explosive anger or blushing, is seen as a manifestation of metal poisoning. It can be seen with other findings such as
tremors progressing to choreiform movements of limbs and gingivostomatitis. Which of the following metals may be
responsible for these manifestations?

A. Lead

B. Mercury

C. Iron

D. Copper

E. Arsenic

549. The findings of congestive cardiomyopathy, pulmonary edema (cardiogenic and non-cardiogenic), pancytopenia with
basophilic stippling of erythrocytes, and ascending sensorimotor peripheral neuropathy with acute exposure, and the
appearance of Aldrich-mees lines months after the acute exposure are consistent with poisoning with which of the following
metals?

A. Lead

B. Mercury

C. Iron

D. Copper

E. Arsenic

550. Chronic exposure with this metal has been linked with malignancies of the lung, skin, and the urinary bladder even at
doses not high enough to cause either acute or chronic toxicities. Which toxic chronic exposures on the hand, important
findings include a “raindrop” pattern of hyperpigmentation and hyperkeratosis of the palms and soloes,and a stocking-glove
pattern of sensorimotor neuropathy?

A. Lead

Downloaded by ;%'%SiLeNt'%; (jemribsagpatan123@gmail.com)


lOMoARcPSD|5190755

B. Mercury

C. Iron

D. Copper

E. Arsenic

551. Which of the following manifestations is most consistent with acute overdose of Iron, particularly in children?

A. Hemorrhagic gastroenteritis

B. Acute hepatic necrosis

C. Acute pulmonary edema

D. Encephalopathy and convulsions

E. Acute Gingivostomatitis

552. Which of the following is/are correct indications for the use of Dimercaprol?

I. Single-agent treatment of acute poisoning with inorganic or elemental mercury

II. Single-agent treatment of acute arsenic poisoning

III. Combination treatment with edentate calcium disodium for severe lead poisoning

A. I only
B. II only
C. I and III
D. II and III
E. I, II and III

553. Penicillamine, as a chelating agent, is primarily used for the treatment of poisoning with which of the following
metals?

A. Lead

B. Mercury

C. Iron

D. Copper

E. Arsenic

554. Deferoxamine is primarily indicated for the treatment of poisoning with which of the following metals?

A. Lead

B. Mercury

C. Iron

D. Copper

E. Arsenic

555. Hemodialysis as an intervention for poisoning is indicated for which of the following drugs?

A. Morphine

B. Digoxin

C. Diazepam

D. Propranolol

E. Ethylene glycol

556. The use of activated charcoal as a means of eliminating a drug taken in overdose is applicable for which of the
following drugs?

A. Carbamazepine

B. Ethanol

C. Ferrous sulfate

D. Lithium carbonate

E. Potassium chloride

Downloaded by ;%'%SiLeNt'%; (jemribsagpatan123@gmail.com)


lOMoARcPSD|5190755

557. Following a non-accidental ingestion of Acetaminophen tablets, what minimum serum acetaminophen level taken 4
hours after ingestion will indicate a high risk of liver injury?

A. > 10mg/ml

B. > 50mg/L

C. > 1000mg/L

D. > 1500 mg/L

E. >2000mg/L

558. In the management of Quinidine-like cadiotoxicity associated with an overdose of a tricyclic antidepressant, which of
the following is the most useful intervention?

A. Physostigmine

B. Lidocaine

C. Quinidine

D. Sodium bicarbonate

E. Magnesium sulfate

559. Seizures as a manifestation of poisoning can be seen as a common component of the toxidrome of which of the
following drugs?

I. Atropine

II. Methampethamine

III. Tricyclic antidepressants

A. I only
B. II only
C. I and III
D. II and III
E. I, II and III

560. Which of the following agents can cause a state of drunkenness followed a day later by severe anioin gap metabolic
acidosis?

A. Ethylene glycol

B. Ethanol

C. Gamma-hydroxybutyrate

D. Lorazepam

E. Diazepam

561. The volume of Distribution of a drug may determine the efficacy of hemodialysis or hemoperfusion as a means of
facilitating elimination of the drug from the body. Which of the following drugs or drug groups have a high volume of
distribution which makes hemodialysis ineffective therapeutic option in cases of poisoning?

I. Antipsychotics

II. Antidepressants

III. Antimalarials

A. I only
B. II only
C. I and III
D. II and III
E. I, II and III

562. Constriction of pupils (miosis) is seen with poisoning or overdoses with which of the following agents?

A. Opoids

B. Tropane alkaloids

C. Amphetamines

D. LSD

E. Cocaine

563. Which of the following agents can cause a high anion gapmetabolic acidosis?

Downloaded by ;%'%SiLeNt'%; (jemribsagpatan123@gmail.com)


lOMoARcPSD|5190755

I. Methanol

II. Ethanol

III. Metformin

A. I only
B. II only
C. I and III
D. II and III
E. I, II and III

564. What is the preferred treatment for severe Verapamil overdose?

A. Propranolol

B. Sodium bicarbonate

C. Epinephrine

D. Calcium gluconate

E. Naloxone

565. Which of the following is most appropriate treatment for the management of cardiac manifestations of theophylline or
Caffeine overdose?

A. Verapamil

B. Esmolol

C. Digoxin

D. Magnesium sulfate

E. Adenosine

566. Which of the following agents is most useful for the management of acetaminophen toxicity?

A. S-carboxymethylcysteine

B. N-acetylcysteine

C.Atropine

D. Ethanol

E. Oxygen

567. Which of the following agents may be useful for the treatment of poisoning due to methanol?

I .Ethanol

II. Glucagon

III. Fomepizole

A. I only
B. II only
C. I and III
D. II and III
E. I, II and III

568. Which of the following agents may be useful for the treatment of ethylene glycol poisoning?

I .Ethanol

II. Glucagon

III. Fomepizole

A. I only
B. II only
C. I and III
D. II and III
E. I, II and III

569. Royal jelly, a popular nutritional supplement form honeybee has been reported to cause which of the following
adverse effects?

A. Hepatotoxicity

B. CNS depression

Downloaded by ;%'%SiLeNt'%; (jemribsagpatan123@gmail.com)


lOMoARcPSD|5190755

C. CNS stimulation

D. Anaphylaxis

E. Hemorrhagic gastritis

570. Ginkgo biloba is a common additive in a number of nutritional supplements. Which subset of patients should be
cautioned against the use ginkgo-containing products?

I. Patients receiving anticoagulation therapy

II. Patients with Alzheimers Dementia

III. Patients with history of seizures

A. I only
B. II only
C. I and III
D. II and III
E. I, II and III

571. Ginseng (Panax ginseng) is a popular additive in nutritional supplement due to its ability to improve physical and
mental performance. Which subset/s of patients must be advised regarding the potential adverse effect of Ginseng on their
disease or when taken concurrently with their medications?

I. Patients with Bronchial asthma

II. Patients taking warfarin

III. Patients with hypertension

A. I only
B. II only
C. I and III
D. II and III
E. I, II and III

572. Delayed neurotoxicity associated with exposure to organophosphate characterized by polyneuropathy, paralysis and
axonal degeneration has been attributed to what effect of these toxicants?

A. Inhibition of the plasma butyrylcholinesterase

B. Inhibition of release of Acetylcholine

C. Inhibition of the RBC acetylcholinesterase

D. Direct inhibition of myelin sheath function

E. Inhibition of the neuropathy target esterase

573. Which of the following statements is/are correct regarding the toxicology of the carbamate insecticides?

I. Inhibition of the cholinesterase enzyme is reversed easily with the use of pralidoxime

II. The clinical approach to management is similar to organophosphate poisoning

III. They are considered as persistent toxicants with great impact in the environment

A. I only
B. II only
C. I and III
D. II and III
E. I, II and III

574. Which of the following mechanisms explains the toxicity of the chlorinated hydrocarbon insecticides like DDT?

A. inhibit the ligand-gated sodium channels

B. Inhibit the enzyme acetylcholinesterase

C. Stimulate the GABA-A receptor

D. Inhibit the inactivation of sodium channels

E. Promote outward potassium transport

575.Toxidromes of poisoning with chlorinated hydrocarbons primarily consists of which of the following manifestations?

A. CNS stimulation

B. CNS depression

C. Bone marrow suppression

Downloaded by ;%'%SiLeNt'%; (jemribsagpatan123@gmail.com)


lOMoARcPSD|5190755

D. Cardiac rhythm disturbance

E. Hepatic failure

576. Which of the following agents is/are considered as botanical insecticide/s (derived from botanical sources)?

I. Nicotine

II. Rotenone

III. Pyrethrum

A. I only
B. II only
C. I and III
D. II and III
E. I, II and III

577. What is the minimum fatal dose of nicotine, equivalent to 2 sticks of cigarettes?

A. 10mg

B. 20 mg

C. 40mg

D. 80 mg

E. 160 mg

578. Which of the following findings is/are consistent with acute nicotine toxicity?

I. Hypertension

II. Peripheral vasodilatation

III. Cardiac arrhythmia

A. I only
B. II only
C. I and III
D. II and III
E. I, II and III

579. Which of the following describe/s the appropriate management of acute nicotine toxicity?

I. Anticonvulsant therapy mainly involves the use of benzodiazepine like Diazepam

II. Epinephrine is given to increase the blood pressure

III. Neostigmine is given to counteract the neuromuscular blockade

A. I only
B. II only
C. I and III
D. II and III
E. I, II and III

580. Paraquat is described as having a toxicity rating of 4. What are the probable human lethal dosages with this rating?

A. 5,000-50,000mg/kg body weight

B. 500- 5,000mg/kg body wt

C. 50- 500mg/kg body wt

D. 5-50mg/kg body wt

E. not more than 5 mg/kg body wt

581. Parathion has a toxicity rating of 6. What are the probable human lethal dosages with this rating?

A. 5,000-50,000mg/kg body weight

B. 500- 5,000mg/kg body wt

C. 50- 500mg/kg body wt

D. 5-50mg/kg body wt

E. not more than 5 mg/kg body wt

Downloaded by ;%'%SiLeNt'%; (jemribsagpatan123@gmail.com)


lOMoARcPSD|5190755

582. The insecticide Azinphos-methyl can cause death when ingested at a dose of 20 mg/kg body wt. What is its toxicity
rating?

A.1

B. 2

C. 5

D. 5

E. 6

583. Which of the following toxicants is known to accumulate slowly in the lungs by an active process and causes lung
edema, alveolitis, and progressive pulmonary fibrosis days to weeks after an exposure?

A. Paraquat

B. Scopolamine

C. Parathion

D. Dioxin

E. Nicotine

584.Yusho disease documented in Japan in the 1960’s is associated with exposure to which of the following toxicants?

A.Polychlorinated biphenyls

B. Paraquat

C. Nicotine

D. Aconite

E. Mercury

585. What is considered as the most common mode of exposure to poisons?

A. inhalation

B. Dermal exposure

C. Ingestion

D. Ophthalmic

E. Bites

586. In the initial treatment of poisoning at home, which of the following instructions is correct to give regarding inducing
vomiting as an initial intervention?

A. Vomiting can be induced safely with either Extract of Ipecac syrup

B. for adults, use 2 tablespoonfuls of Ipecac syrup followed by a glassful of water

C. For children, less than 1 year old, use one tablespoonful of Ipecac followed by a glassful of water

D. If the patient does not vomit with Ipecac, use salt water or mustard as an alternative

E. If the patient does not vomit with Ipecac, try other techniques like tickling the back of the throat of your finger

587. Which of the following situations will contraindicate induction of vomiting as an intervention in cases of poisoning?

I. Patient has ingested petroleum products

II. Patient is less than 1 year old

III. Patient is having convulsion

A. I only
B. II only
C. I and III
D. II and III
E. I, II and III

588. Hyperbaric oxygen supplementation may be necessary intervention if no response is seen with 100% oxygen
supplementation in cases of poisoning with which of the following toxicants?

I. carbon monoxide

II. Nitrobenzene

Downloaded by ;%'%SiLeNt'%; (jemribsagpatan123@gmail.com)


lOMoARcPSD|5190755

III. Cyanide

A. I only
B. II only
C. I and III
D. II and III
E. I, II and III

589. Milk may be used to dilute ingested toxicants. Which of the following ingested poisons may be manage initially by the
administration of milk?

I. Hypochlorite

II. Corrosive acids

III. Fluoride

A. I only
B. II only
C. I and III
D. II and III
E. I, II and III

590. Alteration of pH can alter renal elimination of certain toxins. Which of the following toxicants may be safely eliminated
by either acidification or alkalinization of urine?

I. Phencyclidine

II. Salicylates

III. Amphetamines

A. I only
B. II only
C. I and III
D. II and III
E. I, II and III

591. Which of the following rugs may be useful to manage complication of exposures to nitrogen oxides and nitrobenzene?

A. Methylene blue

B. Ethanol

C. Calcium gluconate

D. Activated charcoal

E. Penicillamine

592. Toxidrome consisting of CNS depression followed by severe metabolic acidosis, blindness with initial “snowstorm-like”
visual disturbance, and coma is most consistent with which of the following toxicants?

A. Nitrates

B. Warfarin

C. Ethanol

D. Methanol

E. Ethylene glycol

593. The manifestations of oxalate poisoning are best managed with which of the following antidotes?

A. Calcium gluconate

B. Fomepizole

C. Glucagon

D. Sodium bicarbonate

E. Atropine

594. A 20 year old male patient is admitted after ingesting 20 tablets of 300mg Isoniazid tablet. Which of the following
statements is/are correct regarding the management of this patient?

I. Administer a total of 6000mg pyridoxine in divided doses by slow IV

II. Give activated charcoal slurry

III. Given benzodiazepines to control seizures

Downloaded by ;%'%SiLeNt'%; (jemribsagpatan123@gmail.com)


lOMoARcPSD|5190755

A. I only
B. II only
C. I and III
D. II and III
E. I, II and III

595. Historically, a “universal antidote” was employed in the management of poisoning. Recent studies have shown that
the two of the components of this so-called universal antidote have no significant efficacy. What are the components of this
so-called “universal antidote”?

A. activated charcoal, calcium oxide, tannic acid

B. activated charcoal, magnesium oxide, tannic acid

C. activated charcoal, milk of magnesia, milk

D. Milk of magnesia, tannic acid, strong tea

E. activated charcoal, milk, ipecac

596. Which of the following interventions is/are applicable in cases of acute opioid overdose?

I. Activated charcoal dose not have a role in the management of opoid poisoning

II. Naloxone can be administered IV, IM, or SC

III. If respiratory depression develops, 2mg of naloxone is administered and may be repeated as necessary

A. I only
B. II only
C. I and III
D. II and III
E. I, II and III

597. A patient is given physostigmine to manage poisoning with anti-muscarinic agent. Which of the following is/are
possible complications of therapy with physostigmine?

I. Cholinergic crisis

II. Seizures

III. Asystole

A. I only
B. II only
C. I and III
D. II and III
E. I, II and III

598. In the management of Mercury Poisoning, which of the following is/are important considerations?

I. dimercaprol is not used for elemental and organic mercury poisoning as the drug may redistribute mercury from tissues
to brain

II. Chelation from unithiol may diminished nephrotoxicity that may result from acute exposure to inorganic mercury

III. In acute exposure, the efficacy of chelation therapy diminishes with time since exposure

A. I only
B. II only
C. I and III
D. II and III
E. I, II and III

599. Dimercaptopropanesulfonic acid is also known as ______?

A. Dimercaprol

B. Succimer

C. D-dimethylcysteine

D. unithiol

E. Deferoxamine

600. What is the mechanism of action of Glucagon that makes the drug useful for the management of overdoses or
poisoning with drugs that directly inhibit myocardial contractility?

A. glucagon directly stimulates calcium release from the sarcoplasmic reticulum

B. Glucagon directly binds to beta adrenergic receptors leading to an increase cAMP

Downloaded by ;%'%SiLeNt'%; (jemribsagpatan123@gmail.com)


lOMoARcPSD|5190755

C. Glucagon binds to glucagons receptors that stimulates production of IP3 and cause an increase in intracellular calcium

D. Glucagon binds to glucagons receptors leading to generation of cAMP

E. Glucagon increases cAMP levels by inhibiting the enzyme phosphodiesterase

f.

Downloaded by ;%'%SiLeNt'%; (jemribsagpatan123@gmail.com)

You might also like